Tests in clinical pharmacology for students. Pre-examination test in pharmacology

Clinical pharmacology test

Gee Test preparation system oldkyx.com

List of questions on Clinical pharmacology

1. The half-life of a drug is:
1) [-] the time to reach the maximum concentration of the drug in plasma;

2) [-] the time during which the drug reaches the systemic circulation;

3) [-] the time during which the drug is distributed in the body;

4) [+] the time during which the concentration of the drug in plasma decreases by 50%;

5) [-] the time during which half of the administered dose reaches the target organ.

2. The breadth of therapeutic action is:
1) [-] therapeutic dose of the drug;

2) [-] the ratio of drug concentration in an organ or tissue to its concentration in blood plasma;

3) [+] the range between the minimum therapeutic and minimum toxic concentrations of the drug in plasma;

4) [-] percentage of non-protein-bound drug;

5) [-] range between the minimum and maximum therapeutic drug concentrations.

3. Competitive receptor drugs include:
1) [-] NSAIDs (non-steroidal anti-inflammatory drugs);

2) [+] β-blockers;

3) [-] loop diuretics;

4) [-] nitrates;

5) [-] fluoroquinolones.

4. The function of the liver and kidneys should be considered when prescribing the following drugs:
1) [-] lipophilic, forming inactive metabolites;

2) [+] lipophilic, forming active metabolites;

3) [-] hydrophilic;

4) [-] hepatotoxic;

5) [-] nephrotoxic.

5. The selectivity of the action of a medicinal substance depends on:
1) [-] half-life;

2) [-] method of reception;

3) [-] connection with the protein;

4) [-] volume of distribution;

5) [+] doses.

6. Saturation kinetics is characterized by:
1) [+] an increase in the half-life of the administered dose with unchanged clearance;

2) [-] the rate of elimination is proportional to the concentration of the drug in plasma and dose;

3) [-] half-life is not proportional to the administered dose.

7. Factor determining the need to recalculate the regimen of drug administration in chronic renal failure:
1) [-] high lipophilicity of the drug;

2) [-] low connection with plasma proteins;

3) [-] the presence of active tubular excretion pathway systems;

4) [+] high degree of excretion unchanged.

8. What drugs pass through the BBB more easily?
1) [-] with high solubility in water;

2) [+] with high solubility in fats;

3) [-] exhibiting the properties of weak acids;

4) [-] exhibiting the properties of weak bases;

5) [-] with a weak connection with plasma proteins.

9. In which case is more complete absorption?
1) [-] absorption from the stomach of a drug exhibiting the properties of a weak base;

2) [-] absorption from the small intestine of a drug exhibiting the properties of a weak acid;

3) [+] absorption from the small intestine of a drug that exhibits the properties of a weak base.

10. The concept of "presystemic metabolism" includes:
1) [+] biotransformation of drugs in the liver during the first passage and in the intestine;

2) [-] biotransformation of drugs in the intestine;

3) [-] biotransformation of drugs in the liver during the first passage and in the kidneys;

4) [-] biotransformation of drugs in the liver, kidneys and intestines.

11. The following group of side effects is strictly dose-dependent:
1) [-] pharmaceutical;

2) [-] pharmacogenetic;

3) [-] allergic;

4) [-] mutagenic;

5) [+] withdrawal syndrome.

12. Define a group of drugs with a narrow therapeutic index:
1) [-] β-blockers;

2) [-] penicillins;

3) [+] cardiac glycosides;

4) [-] ACE inhibitors;

5) [-] powerful diuretics.

13. Carrying out drug monitoring is desirable in the treatment of the following group of drugs:
1) [+] anticonvulsants;

2) [-] β2-symptom mimetics;

3) [-] penicillins;

4) [-] glucocorticoids;

5) [-] M-cholinolytics.

14. Delayed include the following group of side effects:
1) [-] toxic;

2) [-] development of drug dependence;

3) [-] pharmacogenetic;

4) [+] carcinogenic;

5) [-] withdrawal syndrome.

15. The development of asystole is possible with the combination of propranolol with:
1) [-] phenobarbital;

2) [-] furosemide;

3) [+] verapamil;

4) [-] phenytoin;

5) [-] ranitidine.

16. The risk of toxic effects increases when gentamicin is combined with:
1) [+] furosemide;

2) [-] penicillin;

3) [-] methylxanthines;

4) [-] macrolides;

5) [-] glucocorticoids.

17. The risk of unwanted pregnancy increases when oral contraceptives are combined with:
1) [-] hypotensive;

2) [-] vitamin C;

3) [-] alcohol;

4) [+] tetracycline;

5) [-] glucocorticoids.

18. In kidney pathology, the following changes in the pharmacokinetics of drugs occur, except:
1) [-] impaired renal excretion;

2) [-] increase in the concentration of drugs in the blood plasma;

3) [-] decrease in plasma protein binding;

4) [-] increase in T1 / 2;

5) [+] decrease in bioavailability.

19. Cirrhosis of the liver is caused by the following changes in the pharmacokinetics of drugs, except:
1) [-] decrease in first pass metabolism;

3) [-] increase in T1 / 2;

4) [-] increase in bioavailability;

5) [+] reduction in the volume of distribution.

20. In heart failure, the following changes in the pharmacokinetics of digoxin are observed, except:
1) [-] decrease in absorption in the gastrointestinal tract by 30%;

2) [-] decrease in binding to plasma proteins;

3) [+] enhancing metabolism in the liver;

4) [-] decrease in renal excretion;

5) [-] increase in Т1/2.

21. Alcohol in a single dose of large doses leads to:
1) [-] increased drug absorption;

3) [+] slowing down the metabolism in the liver;

4) [-] decrease in renal excretion;

5) [-] increase in T1 / 2.

22. Nicotine leads to:
1) [-] decrease in drug absorption;

2) [-] increase in the volume of drug distribution;

3) [-] decrease in connection with the plasma protein;

4) [+] increased metabolism in the liver;

5) [-] increased renal excretion of drugs.

23. The usual form of drug release is characterized by:
1) [-] nitrong;

2) [-] sustak-mite;

3) [+] nitrosorbide;

4) [-] nifedipine-GITS;

5) [-] verapamil SR.

24. To stop an attack of angina pectoris, a sublingual tablet dosage form is used:
1) [-] nitrong;

2) [-] sustak;

3) [+] nitrosorbide;

4) [-] atenonolol;

5) [-] verapamil SR.

25. To prevent the development of tolerance with regular use of nitrates, the nitrate-free interval should be:
1) [-] 2-4 hours;

2) [-] 4-6 hours;

3) [-] 6-8 hours;

4) [+] 8-12 hours.

26. To enhance the antianginal effect, the safest combination is:
1) [-] verapamil + propranolol;

2) [-] verapamil + atenolol;

3) [-] verapamil + metoprolol;

4) [+] verapamil + isosorbide dinitrate;

5) [-] verapamil + diltiazem.

27. Methods for assessing the antianginal efficacy of the drug are all of the following, except:
1) [-] Holter ECG monitoring;

2) [+] monitoring of daily blood pressure;

3) [-] stress echo;

4) [-] treadmill test;

5) [-] VEM samples.

28. In a patient with angina pectoris in combination with arterial hypertension, the following drugs have an advantage:
1) [-] nitrates;

29. In angina pectoris, the drugs of choice are drugs of the following class:

2) [+] blockers of β-adrenergic receptors;

3) [-] α-adrenergic blockers;

4) [-] imidazoline receptor agonists;

5) [-] angiotensin II receptor blockers.

30. In vasospastic angina, the drugs of choice are drugs of the following class:
1) [-] histamine receptor blockers;

2) [-] blockers of β-adrenergic receptors;

3) [-] α-adrenergic blockers;

4) [+] calcium channel blockers;

5) [-] angiotensin II receptor blockers.

31. An adequate method for monitoring the effectiveness and safety of antihypertensive therapy is:
1) [-] daily ECG monitoring;

2) [+] daily monitoring of blood pressure;

3) [-] one-time blood pressure measurements;

4) [-] measurement of respiratory function;

5) [-] dynamics of the QT interval on the ECG.

32. Select an undesirable effect that is not characteristic of verapamil:
1) [-] bradycardia;

2) [-] constipation;

3) [-] development of AV blockade;

4) [-] swelling of the legs and feet;

5) [+] bronchospasm.

33. For the treatment of arterial hypertension, the drug of first choice in a patient with chronic heart failure is:
1) [+] enalapril;

2) [-] verapamil;

3) [-] clonidine;

4) [-] prazosin;

5) [-] nifedipine.

34. Specify an antihypertensive drug that increases the activity of the sympathoadrenal system:
1) [+] nifedipine;

2) [-] clonidine;

3) [-] captopril;

4) [-] metoprolol;

5) [-] irbesartan.

35. In case of arterial hypertension in combination with sinus tachycardia, preference should be given to:
1) [-] calcium channel blockers of dihydropyridine derivatives;

2) [-] loop diuretics;

3) [+] β-blockers;

4) [-] α-blockers;

5) [-] thiazide diuretics.

36. α1-blockers are the drugs of choice for the treatment of arterial hypertension:
1) [-] in patients with liver disease;

2) [-] in patients with rhythm disturbances;

3) [+] in elderly men with prostate adenoma and difficult urination;

4) [-] in patients with angina pectoris;

5) [-] in patients with a history of myocardial infarction.

37. For the treatment of arterial hypertension in patients with bronchial asthma, the following cannot be used:
1) [-] calcium channel blockers;

2) [-] angiotensin II receptor antagonists;

3) [-] α1-blockers;

4) [+]β-blockers;

5) [-] diuretics.

38. Patients with arterial hypertension and a history of myocardial infarction should first of all be prescribed:
1) [+] β-blockers;

2) [-] diuretics;

3) [-] calcium channel blockers;

4) [-] imidazoline receptor agonists;

5) [-] α1-blockers.

39. First choice drugs in patients with arterial hypertension and severe peripheral arterial stenosis are:
1) [-] non-selective β-blockers;

2) [+] calcium antagonists;

3) [-] diuretics;

4) [-] angiotensin II receptor blockers;

5) [-] α2-adrenergic receptor agonists.

40. List the groups of drugs that improve the prognosis of patients with CHF:
1) [-] ACE inhibitors;

2) [-] β-blockers;

3) [-] angiotensin II receptor blockers;

4) [-] spironolactone;

41. List drugs that have a direct positive inotropic effect:
1) [-] digoxin;

2) [-] dopamine;

3) [-] amrinone;

4) [-] levosimendan;

5) [+] all listed drugs.

42. List β-blockers that have proven efficacy in the treatment of patients with CHF:
1) [-] atenolol;

2) [-] propranolol;

3) [+] carvedilol;

4) [-] sotalol;

43. Specify the indications for the appointment of spironolactone:
1) [-] the level of potassium in the blood> 5.5 mmol/l;

2) [-] swelling of the legs and feet;

3) [+] heart failure IV FC according to the NYHA classification;

4) [-] creatinine clearance less than 30 ml/min;

44. Dose titration of ACE inhibitors and β-blockers in patients with CHF implies:
1) [-] the beginning of therapy with the minimum dose of the drug;

2) [-] increase in the dose of the drug every 2 weeks;

3) [-] achievement of the target dose of the drug;

4) [-] reduction in the number of hospitalizations and an increase in the patient's life expectancy;

5) [+] all the listed features.

45. Indications for the appointment of amlodipine in CHF:
1) [-] congestive heart failure;

2) [+] uncontrolled blood pressure figures;

3) [-] history of myocardial infarction;

4) [-] rhythm disturbances;

5) [-] all of the listed features.

46. ​​Principles of diuretic therapy in CHF:
1) [-] the appointment of diuretics in heart failure II-IV FC according to the NYHA classification;

2) [-] weight loss by 0.5-1.0 kg per day;

3) [-] blood pressure control;

4) [-] control of the level of potassium in the blood;

5) [+] all of the above.

47. Specify the preferred route of drug administration for congestive heart failure:
1) [-] sublingual;

2) [-] rectal;

3) [+] parenteral;

4) [-] oral;

5) [-] all of the listed routes of administration.

48. Effects of an ACE inhibitor in patients with CHF:
1) [-] the effect of an ACE inhibitor on mortality depends on the duration of treatment;

2) [-] the reduction in the risk of death is more pronounced in patients with higher FC;

3) [-] the presence of a dose-dependent effect of an ACE inhibitor in patients with heart failure;

4) [+] all of the above.

49. List the drugs that increase the duration of the action potential:
1) [-] quinidine;

2) [-] procainamide;

3) [-] amiodarone;

4) [-] digoxin;

5) [+] all listed drugs.

50. Drugs that prolong the QT interval:
1) [-] clindamycin;

2) [-] amiodarone;

3) [-] cotrimoxazole;

4) [-] quinidine;

5) [+] all listed drugs.

51. Drugs that prolong the PQ interval:
1) [-] lidocaine;

2) [-] quinidine;

3) [-] disopyramide;

4) [+] digoxin;

5) [-] all of the listed drugs to the appointment of antiarrhythmic drugs.

52. Indications for prescribing antiarrhythmic drugs are:
1) [-] frequent rhythm disturbances;

2) [-] ventricular extrasystoles - 6 per minute;

3) [-] violation of the rhythm of high gradations;

4) [+] violation of hemodynamics;

5) [-] all of the above.

53. List drugs that have anticholinergic side effects:
1) [-] amiodarone;

2) [-] verapamil;

3) [-] lidocaine;

4) [+] quinidine;

5) [-] all listed drugs.

54. List drugs that increase the threshold of fibrillation:
1) [-] cordarone;

2) [-] bretylium tosylate;

3) [-] sotalol;

4) [-] propranolol;

5) [+] all listed drugs.

55. List the drugs prescribed for the prevention of arrhythmias in WPW syndrome:
1) [-] digoxin;

2) [-] diltiazem;

3) [+] amiodarone;

4) [-] procainamide;

5) [-] all listed drugs.

56. Indications for the restoration of the rhythm with a constant form of atrial fibrillation:
1) [-] frequent episodes of tachysystole;

2) [-] weakness of the sinus node;

3) [+] history of thromboembolism;

4) [-] inefficiency of ongoing drug therapy;

5) [-] all of the above.

57. The drug of choice for the treatment of paroxysms of ventricular tachycardia:
1) [-] lidocaine;

2) [-] procainamide;

3) [-] bretylium;

4) [-] propafenone;

5) [+] all listed drugs.

58. Side effects of amiodarone:
1) [-] photosensitivity;

2) [-] dysfunction of the thyroid gland;

3) [-] dry cough;

4) [-] transient increase in the activity of hepatic aminotransferases;

5) [+] all the listed effects.

59. Indications for the use of adenosine:
1) [-] paroxysm of atrial fibrillation;

2) [+] reciprocal supraventricular tachycardias;

3) [-] ventricular tachycardia;

4) [-] extrasystole;

5) [-] all of the above.

60. Method for evaluating the effectiveness of antiarrhythmic therapy in outpatient practice:
1) [+] Holter ECG monitoring;

3) [-] tests with physical activity;

4) [-] EFI (electrophysiological study);

5) [-] all listed methods.

61. Inhaled glucocorticoids include:
1) [-] hydrocortisone;

2) [+] beclomethasone;

3) [-] prednisolone;

4) [-] polcortalon;

5) [-] dexamethasone.

62. Long-acting selective β2-agonists include:
1) [-] fluticasone;

2) [+] salmeterol;

3) [-] salbutamol;

4) [-] fenoterol;

5) [-] terbutaline.

63. To stop an attack of bronchial asthma, the following is used:
1) [-] tiotropium bromide;

2) [-] teopek;

3) [-] sodium cromoglycate;

4) [+] salbutamol;

5) [-] budesonide.

64. Long-acting anticholinergics include:
1) [-] ipratropium bromide;

2) [-] sodium cromglycate;

3) [+] tiotropium bromide;

4) [-] oxitropium bromide;

5) [-] triamcinolone acetonide.

65. A side effect of inhaled glucocorticosteroids is:
1) [-] headache;

2) [-] obesity;

3) [+] oral candidiasis;

4) [-] diabetes mellitus;

5) [-] polyuria.

66. Mucolytic drugs include:
1) [-] codeine;

2) [-] sodium cromoglycate;

3) [+] acetylcysteine;

4) [-] salmeterol;

5) [-] theophylline.

67. With simultaneous use increases the concentration of theophylline in the blood:
1) [+] ofloxacin;

2) [-] penicillin;

3) [-] ceftriaxone;

4) [-] gentamicin;

5) [-] biseptol.

68. With simultaneous use, it reduces the concentration of theophylline in the blood:
1) [-] pefloxacin;

2) [-] cimetidine;

3) [+] rifampicin;

4) [-] erythromycin;

5) [-] ampioks.

69. Bronchodilators do not include:
1) [-] methylxanthines;

2) [-] anticholinergics;

3) [-] sympathomimetics;

4) [+] leukotriene receptor blockers.

70. The drug of basic therapy for chronic obstructive bronchitis is:
1) [+] tiotropium bromide;

2) [-] nedocromil sodium;

3) [-] fenoterol;

4) [-] montelukast;

5) [-] aminofillin.

71. Tachycardia as a side effect develops when taking all of the following drugs, except:
1) [-] salbutamol;

2) [-] isoproterinol;

3) [-] fenoterol;

4) [-] theophylline;

5) [+] ipratropium bromide.

72. Combined drugs for the treatment of bronchial asthma do not include:
1) [-] ditek;

2) [-] seretide;

3) [-] symbicort;

4) [+] beclazone;

5) [-] berodual.

73. Inhaled glucocorticosteroid has the most pronounced anti-inflammatory effect:
1) [-] beclamethasone dipropionate;

2) [-] budesonide;

3) [-] triamcinolone acetonide;

4) [+] fluticasone propionate;

5) [-] flunisolide.

74. The drug of choice in the presence of infections caused by methicillin-resistant strains of Staphylococcus aureus is:
1) [-] azithromycin;

2) [-] metronidazole;

3) [-] gentamicin;

4) [+] linezolid;

5) [-] cefuroxime.

75. Select a group of antibacterial drugs for the treatment of infections caused by intracellular pathogens:
1) [+] macrolides;

2) [-] penicillins;

3) [-] aminoglycosides;

4) [-] cephalosporins;

5) [-]sulfonamides.

76. Specify the group of antibacterial drugs with the highest antianaerobic activity:
1) [-] glycopeptides;

2) [-] aminopenicillins;

3) [-] tetracyclines;

4) [-] aminoglycosides;

5) [+] nitroimidazoles.

77. All of the listed antibacterial drugs are nephrotoxic, except:
1) [-] gentamicin;

2) [-] carbenicillin;

3) [+] azithromycin;

4) [-] cefazolin;

5) [-] vancomycin.

78. Specify an antibacterial drug, inactive against pneumococcus:
1) [-] azithromycin;

2) [-] penicillin;

3) [-] ceftriaxone;

4) [+] ciprofloxacin;

5) [-] chloramphenicol.

79. Choose a rational combination of antibacterial drugs that has a synergistic effect against gram-positive microorganisms and safety:
1) [-] penicillins + tetracyclines;

2) [-] penicillins + cephalosporins;

3) [-] aminoglycosides + glycopeptides;

4) [+] penicillins + aminoglycosides;

5) [-] penicillins + sulfonamides.

80. The following antibacterial drugs penetrate well through the blood-brain barrier:
1) [-] lincosamides;

2) [-] macrolides;

3) [-] tetracyclines;

4) [-] aminoglycosides;

5) [+] III generation cephalosporins.

81. The drug of choice for lobar pneumonia is:
1) [-] ciprofloxacin;

2) [-] doxycycline;

3) [-] gentamicin;

4) [-] cefotaxime;

5) [+] benzylpenicillin.

82. The drug of choice for tonsillopharyngitis is:
1) [+] cefuroxime axetil;

2) [-] doxycycline;

3) [-] ceftazidime;

4) [-] ofloxacin;

5) [-] furagin.

83. Drugs of choice for biliary tract infections are:
1) [-] aminoglycosides;

2) [-] nitrofurans;

3) [+] III generation cephalosporins;

4) [-] macrolides;

5) [-] natural penicillins.

84. In case of hospital infections occurring in intensive care units, the following combinations of antibacterial drugs are the drugs of choice:
1) [-] ampicillin + gentamicin;

2) [+] ceftazidime + amikacin;

3) [-] cefuroxime + erythromycin;

4) [-] clindamycin + gentamicin;

5) [-] norfloxacin + penicillin.

85. The preferred group of antibacterial drugs in the treatment of chronic prostatitis are:
1) [-] sulfonamides;

2) [-] carbapenems;

3) [-] quinolones;

4) [-] lincosamines;

5) [+] fluoroquinolones.

86. Choose a drug that maximally suppresses the secretion of hydrochloric acid:
1) [-] pirenzepine;

2) [-] cimetidine;

3) [-] misoprostol;

4) [-] antacids;

5) [+] omeprazole.

87. The maximum number of side effects among H2-blockers has:
1) [+] cimetidine;

2) [-] roxatidine;

3) [-] nizatidine;

4) [-] ranitidine;

5) [-] famotidine.

88. Inhibits cytochrome P-450:
1) [-] omeprazole;

2) [-] pirenzepine;

3) [+] cimetidine;

4) [-] famotidine;

5) [-] lansoprazole.

89. Recoil syndrome is caused by:
1) [-] synthetic prostaglandins;

2) [-] antacids;

4) [-] M-cholinolytics;

5) [+] H2 blockers.

90. In duodenal ulcer it is rational to prescribe antacids:
1) [-] before meals;

2) [-] while eating;

3) [+] 1.5-2 hours after eating;

4) [-] 5 hours after eating;

5) [-] regardless of food intake.

91. For the prevention of ulcers caused by NSAIDs, the following are most effective:
1) [-] antacids;

2) [-] H2 blockers;

3) [-] "proton pump" blockers;

4) [+] synthetic prostaglandins;

5) [-] M-cholinolytics.

92. In eradication therapy for the destruction of H. pylori, the following antibiotic is used:
1) [-] carbenicillin;

2) [-] erythromycin;

3) [-] cefoperazone;

4) [+] clarithromycin;

5) [-] chloramphenicol.

93. Of great practical importance is the development of H. pylori resistance to:
1) [-] vancomycin;

2) [+] metronidazole;

3) [-] tetracycline;

4) [-] nitrofurans;

5) [-] cefotaxime.

94. Bactericidal action against H. pylori has:
1) [-] sucralfate (venter);

2) [+] bismuth subcitrate (de-nol);

3) [-] almagel;

4) [-] famotidine;

5) [-] pirenzepine.

95. In eradication therapy for the destruction of H. pylori, the following are used:
1) [-] antacids;

2) [-] synthetic prostaglandins;

3) [+] "proton pump" blockers;

4) [-] M-cholinolytics;

5) [-] glycopeptides.

96. Corticosteroids increase toxicity:
1) [-] theophylline;

2) [+] thiazide diuretics;

3) [-] preparations of gold;

4) [-] antiulcer drugs.

97. The effect of glucocorticoids reduces:
1) [-] cimetidine;

2) [-] aspirin;

3) [-] diclofenac;

4) [-] amiodarone;

5) [+] Rifampicin.

98. The highest mineralocorticoid activity has:
1) [-] polkartolon;

2) [-] prednisolone;

3) [+] hydrocortisone;

4) [-] dexamethasone.

99. After taking NSAIDs, the effect develops faster:
1) [-] anti-inflammatory;

2) [+] analgesic;

3) [-] anticoagulant.

100. The most pronounced analgesic property has:
1) [-] acetylsalicylic acid;

2) [-] ibuprofen;

3) [-] naproxen;

4) [+] paracetamol.

101. The mechanism of development of the ulcerogenic effect of NSAIDs is:
1) [-] increased acidity of gastric juice;

2) [+] decrease in the synthesis of prostaglandins in the gastric mucosa;

3) [-] decreased mucosal repair.

102. The most pronounced anti-inflammatory property has:
1) [+] phenylbutazone;

2) [-] metamizole;

3) [-] piroxicam;

4) [-] paracetamol;

5) [-] ibuprofen.

103. An early side effect of glucocorticoids is:
1) [-] cataract;

2) [-] myopathy;

3) [-] osteoporosis;

4) [-] Cushingoid syndrome;

5) [+] steroid diabetes.

104. Specify the effect that is not typical for corticosteroids:
1) [-] anti-inflammatory;

2) [-] antiallergic;

3) [-] antishock;

4) [-] immunosuppressive;

Preview:

Subject: "PMRS affecting the peripheral nervous system"

Test tasks

1. Adrenaline causes:

Choose one answer.

a.) Reduced oxygen consumption

B.) Hyperglycemia

C.) Inhibition of glycogenolysis

D.) Inhibition of lipolysis

2. Adrenaline is contraindicated in:

Choose one answer.

a.) Thyrotoxicosis

B.) Anaphylactic shock

C.) Heart block

D.) Hypoglycemic coma

3. Ganglioblokator:

Choose one answer.

a.) atropine;

B.) pipecuronium;

C.) pentamine;

D.) succinylcholine (ditilin).

4. Ganglioblockers are used to treat:

Choose one answer.

a.) constipation.

B.) hypertensive crisis;

C.) glaucoma;

D.) urinary retention;

5. The action of the M-XR agonist is blocked:

Choose one answer.

a.) Cytisine

B.) Tubocurarine

C.) Prozerin

D.) Atropine

E.) Pilocarpine

6. To stop the action of competitive muscle relaxants, apply:

Choose one answer.

a.) atropine;

B.) dipyroxime.

C.) neostigmine (prozerin);

7. Selective M-cholinomimetic (agonist of muscarinic cholinergic receptors):

Choose one answer.

a.) Prozerin

B.) Pilocarpine

C.) Cytisine

D.) Physostigmine

E.) Carbacholin

8. Adsorbents include:

Choose one answer.

a.) Starch slime.

B.) Decoction of oak bark;

C.) Tannin;

D.) Activated carbon;

9. All of the following are irritants except:

Choose one answer.

a.) Bismuth nitrate basic;

B.) Menthol.

C.) Purified turpentine oil (turpentine);

D.) Mustard paper;

10.M-anticholinergic:

Choose one answer.

a.) pentamine;

B.) pipecuronium;

C.) succinylcholine (ditilin).

D.) atropine;

11.M-anticholinergics cause the development of mydriasis:

Choose one answer.

a.) increasing the tone of the radial muscle of the iris;

B.) reducing the tone of the circular muscle of the iris;

C.) increasing the tone of the ciliary muscle.

12.M-anticholinergics are used to treat:

Choose one answer.

a.) arterial hypertension;

B.) glaucoma;

C.) gastric ulcer.

D.) myasthenia gravis;

13.M-anticholinergics are contraindicated in:

Choose one answer.

a.) bronchial asthma;

B.) glaucoma;

C.) atrioventricular blockade;

D.) gastric ulcer.

14.M-cholinomimetics, unlike ChE inhibitors, do not affect cholinergic synaptic transmission:

Choose one answer.

a.) at the neuromuscular synapse

B.) from postganglionic axons of the autonomic nerves to the effector (smooth muscle, exocrine glands)

C.) in the CNS

15. Local anesthetics are used in combination with adrenaline, because:

Choose one answer.

a.) the absorption of the anesthetic is accelerated and the local anesthetic effect is enhanced.

B.) the absorption of the anesthetic slows down and the local anesthetic effect increases;

C.) the absorption of the anesthetic slows down and the local anesthetic effect is weakened;

16. Metoprolol is indicated for the treatment of:

Choose one answer.

a.) Atrioventricular block

B.) Bronchial asthma

C.) Benign prostatic hyperplasia

D.) Arterial hypertension

17. The mechanism of action of astringents is due to:

Choose one answer.

a.) sodium channel block;

B.) adsorption of chemical compounds;

C.) coating of mucous membranes with a film that prevents irritation of sensory nerves. d.) coagulation of proteins and the formation of a film that protects the endings of sensory nerves from irritation;

18. The mechanism of action of local anesthetics is due to:

Choose one answer.

a.) Blockage of calcium channels and lengthening of the absolute refractory period;

B.) Block of potassium channels and impossibility of membrane repolarization;

C.) Activation of chloride channels and hyperpolarization.

D.) Block of sodium channels and impossibility of membrane depolarization;

19. Muscle relaxant:

Choose one answer.

a.) scopolamine.

B.) pipecuronium;

C.) atropine;

D.) pentamine;

20. Undesirable side effect of ganglionic blockers:

Choose one answer.

a.) hypertensive crisis;

B.) increased intraocular pressure.

C.) orthostatic collapse;

D.) bronchospasm;

21. Non-selective beta-blocker:

Choose one answer.

a.) Metoprolol

B.) Atenolol

C.) Prazosin

D.) Propranolol

22. Neostigmine (prozerin) is used to treat myasthenia gravis because it improves cholinergic synaptic transmission:

Choose one answer.

a.) In the autonomic ganglion

B.) At the myoneural junction

C.) From postganglionic cholinergic fibers to effector organ cells

23. Norepinephrine increases:

Choose one answer.

a.) Peripheral vascular resistance

B.) Motility of the gastrointestinal tract

C.) Bronchial tone

D.) Heart rate

24. Why is atropine (a tertiary amine) superior to metacin (a quaternary ammonium compound) in CNS action:

Choose one answer.

a.) better distributed throughout the body (> Vd values);

B.) better absorbed into the systemic circulation from the injection site (> bioavailability coefficient);

C.) is more slowly excreted (eliminated) from the body (> T1 / 2 values).

25. Why is galanthamine (a tertiary amine) superior to proserin (a quaternary ammonium compound) in CNS action:

Choose one answer.

a.) Slower excreted (eliminated) from the body (> T1 / 2 values)

B.) Better absorbed into the systemic circulation from the injection site (> bioavailability factor)

C.) Better distributed throughout the body (> Vd values)

26. Prazosin causes:

Choose one answer.

a.) Decreased tone of bronchial smooth muscle

B.) Decreased intraocular pressure

C.) Decrease and weakening of heart contractions

D.) Decreased peripheral vascular resistance

27. Propranolol causes:

Choose one answer.

a.) Decreased gastrointestinal motility

B.) Contraction of the iris muscle (mydriasis)

C.) Decreased bronchial tone

D.) Decreased heart rate

28. Reflex bradycardia causes:

Choose one answer.

a.) Salbutamol

B.) Prazosin

C.) Metoprolol

D.) Carvedilol

E.) Norepinephrine

29. The contractile activity of the myometrium is reduced by:

Choose one answer.

a.) Salbutamol

B.) Carvedilol

C.) Propranolol

D.) Norepinephrine

E.) Metoprolol

30. Means for the treatment of acute vascular insufficiency:

Choose one answer.

a.) Metoprolol

B.) Salbutamol

C.) Dobutamine

D.) Norepinephrine

E.) Propranolol

31. Only for superficial anesthesia is used:

Choose one answer.

a.) Benzocaine (anesthesin).

B.) Bupivacaine;

C.) Procaine (novocaine);

D.) Lidocaine;

32. Cholinomimetics are contraindicated in:

Choose one answer.

a.) Myasthenia gravis

B.) Alzheimer's disease

C.) Bronchial asthma

D.) Xerostomia

E.) Glaucoma

Preview:

Subject: "Chemotherapeutic agents"

Test tasks

1. The mechanism of action of AG on protein synthesis in microbial cells is based on their ability to inhibit:

Choose one answer.

a.) DNA polymerase

B.) RNA polymerase

C.) transpeptidation process

D.) mRNA code reading process

2. Choose an antibiotic - an inhibitor of protein synthesis in a bacterial cell:

Choose one answer.

a.) benzylpenicillin

B.) carbenicillin

C.) ampicillin

D.) gentamicin

3.Choose a definition of "chemotherapy"

Choose one answer.

a.) chemotherapy is the suppression of pathogens on the surface of the human body (skin, mucous membranes)

B) Chemotherapy is the suppression of pathogens in the environment (care items, instruments, patient discharges)

C.) chemotherapy is an effect on the cells of a macroorganism

D.) chemotherapy is the suppression of pathogens in the internal environment of the macroorganism

4. The principles of chemotherapy include the following:

Choose one answer.

a.) all answers are correct

B.) Treatment with antibacterial drugs should be started as soon as possible after the onset of the disease

C.) the drug should be selected taking into account the sensitivity of the pathogen to the chemotherapeutic agent;

D.) the dose of the chemotherapeutic agent should be prescribed taking into account the severity of the disease in the patient

5. Nitrofuran derivatives include:

Choose one answer.

a.) Phthalylsulfathiazole (phthalazole)

B.) nalidixic acid

C.) furazolidone

D.) nitrohexoline

6. Which of the following antibiotics disrupt the synthesis of the cell wall:

Choose one answer.

a.) chloramphenicol

B.) tetracyclines

C.) beta-lactam antibiotics

D.) polymyxins

7. What drugs for the treatment of onychomycosis give the lowest recurrence rate?

Choose one answer.

a.) terbinafine and itraconazole;

B.) amphotericin B and nystatin;

C.) griseofulvin and levorin;

D.) zinc undecylenate and iodine

8. What drugs are effective against respiratory syncytial viruses and influenza viruses?

Choose one answer.

a.) ribavirin, interferon;

B.) oseltamivir, rimantadine

C.) azidothymidine, saquinavir;

D.) acyclovir, famciclovir;

9. Which of the statements correctly reflects one of the general principles of chemotherapy for an infectious disease.

Choose one answer.

B.) Clinical improvement is grounds for discontinuation of therapy

C.) the effectiveness of treatment often does not depend on the duration of antibiotic therapy. d. 3) after clinical improvement, treatment should not be stopped and continued if necessary for another 48-72 hours.

10. What antibiotic is effective in pseudomembranous colitis?

Choose one answer.

a.) dicloxacyclline

B.) furazolidone

C.) vancomycin

D.) ampicillin

11. Which of the following antibiotics belongs to beta-lactam antibiotics:

Choose one answer.

a.) streptomycin

B.) meropenem

C.) tetracycline

D.) polymyxin

12. Which of the chemotherapy drugs belongs to sulfonamides:

Choose one answer.

a.) mencomycin

B.) erythromycin

C.) streptomycin

D.) sulfadimidine

13. Which macrolide has the lowest clearance?

Choose one answer.

a.) erythromycin

B.) azithromycin

C.) clarithromycin

D.) roxithromycin

14. Which oral medication is effective for fungal meningitis (eg, cryptococcal)?

Choose one answer.

a.) amphotericin B;

B.) fluconazole

C.) terbinafine;

D.) ketoconazole;

15. What preparation of benzylpenicillin belongs to biosynthetic:

Choose one answer.

a.) ampicillin

B.) benzylpenicillin-benzathine

C.) azlocillin

D.) carbenicillin

16. What drug is used to treat gastrointestinal candidiasis?

Choose one answer.

a.) clotrimazole

B.) griseofulvin;

C.) nitrofungin;

D.) nystatin;

17. What drug is used to treat systemic mycoses?

Choose one answer.

a.) nystatin;

B.) clotrimazole

C.) amphotericin B;

D.) griseofulvin;

18. Which drug is effective against influenza A and B viruses?

Choose one answer.

a.) rimantadine

B.) acyclovir;

C.) azidothymidine;

D.) oseltamivir;

19. The mechanism of action of sulfonamides is associated with:

Choose one answer.

a) inhibition of COX

B.) inhibition of dihydrofolate reductase;

C.) competitive antagonism with PABA and inhibition of dihydropteroate synthetase

D.) competitive antagonism with GABA

20. The most common complication of beta-lactam antibiotics is

Choose one answer.

a.) arrhythmias

B.) oppression of hematopoiesis

C. 1) allergic reactions

D.) hearing loss

21. Polymyxins are considered as 3rd line (“deep reserve”) drugs because:

Choose one answer.

a.) have low efficiency

B.) widespread resistance to them

C.) due to low antimicrobial activity

D.) due to high organotoxicity

22. When using resorptive sulfonamides, the following side effects are possible:

Choose one answer.

a.) agranulocytosis

B.) all of the above

C.) crystalluria

D.) hemolytic anemia, methemoglobinemia

23. Antiviral agents (PVA) are most effective when treated early, because:

Choose one answer.

a.) PVAs exhibit a vistatic effect;

B.) PVAs exhibit a viricidal effect;

C.) PVAs do not show organotoxicity

D.) PVAs are organotoxic;

24. Specify antiretroviral drugs (for the treatment of HIV infection):

Choose one answer.

a.) arbidol, oseltamivir;

B.) azidothymidine, saquinavir;

C.) acyclovir, famciclovir;

D.) interferon, ganciclovir

25. Specify the mechanism of action of fluoroquinolones:

Choose one answer.

a.) increasing the permeability of the CPM

B.) Inhibition of bacterial wall synthesis

C.) inhibition of PDEase

D.) inhibition of DNA gyrase

26. Specify the drug related to oxazolidinones:

Choose one answer.

a.) linezolid

B.) moxifloxacin

C.) co-trimoxazole

D.) lincomycin

27. Specify an antiherpetic agent:

Choose one answer.

a.) azidothymidine;

B.) acyclovir;

C.) arbidol;

D.) saquinavir

28. What is typical for doxycycline?

Choose one answer.

a.) poorly absorbed from the gastrointestinal tract

B.) reduced bioavailability when taken with food

C.) T1/2 16-24 hours

D.) the main route of excretion through the MVP

Preview:

Subject : "General pharmacology"

Test tasks

1 . Substances with affinity and internal activity are called:

Choose one answer.

a.) antagonists

B.) agonists

2 . The action of substances that developed after its entry into the systemic circulation is called:

Choose one answer.

a.) resorptive

B.) local

C.) side

D.) reflex

3 . What is the name of the action of a substance if it interacts only with functionally unambiguous receptors in a certain localization and does not affect other receptors?

Choose one answer.

a.) reflex

B.) reversible

C.) irreversible

D.) selective

4 . What is the name of the accumulation of drugs in the body, with its repeated administration?

Choose one answer.

a.) tachyphylaxis

B.) material cumulation

C.) idiosyncrasy

D.) sensitization

5 . What is the name of the decrease in the effectiveness of the action of the substance with its repeated administration?

Choose one answer.

a.) tolerance (addictive)

B.) cumulation

C.) idiosyncrasy

D.) addiction

6. What is the name of the phenomenon when drug withdrawal causes mental and somatic disorders associated with dysfunctions of many body systems up to death?

Choose one answer.

a.) withdrawal syndrome

B.) abstinence

C.) sensitization

D.) idiosyncrasy

7. Which of the processes takes place in the biotransformation phase, which is called conjugation?

Choose one answer.

a.) hydrolysis

B.) recovery

C.) acidification

D.) acetylation

8. Which answer is most consistent with the term "receptor"?

Choose one answer.

a.) active groups of macromolecules of substrates with which the medicinal substance interacts

B.) drug-activated transport systems

C.) drug-activated redox enzymes

D.) ion channels of biological membranes, the permeability of which is changed by the medicinal substance

9. What parameter of pharmacokinetics is designated as "T1 / 2":

Choose one answer.

a.) elimination rate constant

B.) half-life (half-life, half-life) of substances

C.) absorption from the injection site of 50% of the substance

D.) total ground clearance

10. Metabolic biotransformation is:

Choose one answer.

a.) interaction with glucuronic acid

B.) the transformation of a substance due to oxidation, reduction, hydrolysis

C.) binding to plasma albumin

D.) methylation and acetylation of substances

11. The volume of drug distribution reflects:

Choose one answer.

a.) the ratio of single and daily doses of the medicinal substance

B.) Hypothetical volume of fluid in which the drug is distributed

C.) Estimated amount of drug reaching the systemic circulation

D.) dose-weight ratio

12.Volume of distribution is low if:

Choose one answer.

a.) the substance is in the plasma, in the interstitial and intracellular fluid and accumulates in the tissues

B.) the substance is in the plasma and in the interstitial fluid

C.) the substance is in the plasma, in the interstitial and intracellular fluid

D.) the substance accumulates in the blood plasma

13. Note the main mechanism of absorption of medicinal substances:

Choose one answer.

a.) pinocytosis

B.) passive diffusion

C.) active transport

D.) filtering

14. Pharmacokinetics includes:

Choose one answer.

a.) biotransformation of drugs in the body

B.) The effect of drugs on the genetic apparatus

C.) complications of drug therapy

D.) Effect of drugs on metabolism in the body

15. What does the concept of pharmacodynamics include?

Choose one answer.

a.) drug metabolism in the body

B.) storage conditions of medicines

C.) Biological effects of drugs

D.) method of drug administration

16. What is included in the concept of "Biotransformation":

Choose one answer.

a.) binding of substances to plasma proteins

B.) accumulation of substances in adipose tissue

C.) a complex of physicochemical and biochemical transformations of a medicinal substance aimed at removing it from the body

D.) drug accumulation in muscle tissue

17. What is called the internal activity of a substance?

Choose one answer.

a.) the ability of a substance to recognize it when interacting with a receptor

B.) the ability of a substance to interact with transport systems

C.) the ability of a substance, when interacting with a receptor, to stimulate it and cause a biological effect

D.) the ability of a substance to interact with plasma proteins

18. What does the term "affinity" mean?

Choose one answer.

a.) the affinity of a substance for the transport systems of the body

B.) the affinity of the substance for blood plasma albumins

C.) affinity of drugs for microsomal liver enzymes

D.) the affinity of a substance for a receptor, leading to the formation of a “substance-receptor” complex with it

19.What does the term "bioavailability" mean:

Choose one answer.

a.) degree of binding of substances to plasma proteins

B.) the amount of the substance in the urine relative to the initial dose of the drug

C.) the ability to pass through the blood-brain barrier

D.) the amount of unchanged substance that has reached the blood plasma, relative to the initial dose of the drug

20. What corresponds to the concept of "active transport":

Choose one answer.

a.) invagination of the cell membrane with the formation of a vacuole

B.) transport against a concentration gradient with energy expenditure

C.) transport along a concentration gradient without energy consumption

D. 1) facilitated diffusion

Preview:

Subject : "Immunotropic agents"

Test tasks

1. H1-histamine receptor blockers are used for all of the following indications EXCEPT:

Choose one answer.

a.) urticaria;

B.) bronchial asthma

C.) drug allergy;

D.) seasonal rhinitis;

2. What types of pharmacodynamic effects of glucocorticoids are used in medicine?

Choose one answer.

a.) all are true except 1

B.) hyperglycemic, suppression of the growth zones of the epiphyses;

C.) anti-shock, detoxifying (induction of liver enzymes);

D.) all of the above;

E.) immunosuppressive, anti-allergic, anti-inflammatory;

3. What drugs are most effective as anti-inflammatory agents?

Choose one answer.

a.) broad spectrum antibiotics

B.) NSAIDs;

C.) SPVS;

D.) mast cell membrane stabilizers;

4. What are the acceptable criteria for the effectiveness of insulin therapy?

Choose one answer.

a.) euglycemia, euglucosuria;

B.) euglycemia, aglucosuria;

C.) aglycemia, aglucosuria

D.) normoglycemia, euglucosuria;

5. What are the local undesirable effects of the systematic use of GCS-containing ointments and creams?

Choose one answer.

a.) swelling, hyperemia, soreness;

B.) osteoporosis, hirsutism, dysmenorrhea

C.) hypertrophy, hyperpigmentation, candidiasis;

D.) increased risk of local infections, atrophy, depigmentation;

6. What are the most dangerous side effects of systemic corticosteroids with long-term use?

Choose one answer.

a.) withdrawal syndrome (adrenal insufficiency);

B.) all of the above;

C.) Itsenko-Cushing's syndrome ("Cushingoid");

D.) 1 and 2 are correct.

E.) immunodeficiency state;

7. What are the indications for the use of gestagens?

Choose one answer.

a.) hormone replacement therapy after ovariohysterectomy;

B.) breast cancer, prostate cancer;

C.) dysfunctional uterine bleeding, recurrent miscarriage, endometriosis, contraception;

D.) contraception in women with high hyperdyslipidemia, diabetes mellitus, history of cholestasis

8. What is the most common complication in the treatment with insulin preparations?

Choose one answer.

a.) lipodystrophy;

B.) hypokalemia;

C.) insulin resistance

D.) hypoglycemia;

9. Which GCS drug has low bioavailability when applied topically (on the skin)?

Choose one answer.

a.) budesonide;

B.) fluocinolone acetonide (sinaflan);

C.) prednisolone hemisuccinate

D.) beclamethasone propionate;

10. Which inhaled corticosteroid drug has the lowest risk of systemic adverse effects?

Choose one answer.

a.) prednisolone hemisuccinate

B.) beclamethasone propionate;

C.) budesonide;

D.) fluocinolone acetonide (sinaflan);

11. Which drug belongs to insulin sensitizers?

Choose one answer.

a.) metformin;

B.) humulin

C.) pioglitazone;

D.) acarbose;

E.) glibenclamide;

12. What drug is used for seasonal allergic reactions (hay fever) only as a means of prevention?

Choose one answer.

a.) clemastine;

B.) hydrocortisone;

C.) sodium cromoglycate;

D.) all of the above

13.H1-histamine blockers of the second generation differ from the drugs of the first generation

Choose one answer.

a.) pronounced sedative effect;

B.) antiemetic action

C.) significant M-anticholinergic action;

D.) greater selectivity of action;

14. Oxytocin is characterized by all properties EXCEPT

Choose one answer.

a.) uterine sensitivity is consistently high

B.) effective in small doses as a stimulant;

C.) effective in high doses as a uterotonic;

D.) the sensitivity of the uterus to it increases for childbirth;

15. Indications for the use of cytostatic immunosuppressants include all of the following EXCEPT:

Choose one answer.

a.) autoimmune diseases;

B.) ROT prevention

C.) severe anaphylactic reactions;

16. When hypothyroidism is used as a means of replacement therapy

Choose one answer.

a.) protirelin;

B.) potassium iodide;

C.) thyrotropin

D.) levothyroxine;

17. The use of cytostatic immunosuppressants (methotrexate, fluorouracil, cyclophosphamide) is often complicated

Choose one answer.

a.) leukopenia and infectious syndrome;

B.) allergies and photodermatitis;

C.) bleeding and anemia;

D.) drowsiness and lethargy

18. Thiamazole (Mercazolil) as a means of basic (long-term) treatment is indicated for ...

Choose one answer.

a.) thyroid cancer;

B.) myxedema

C.) nodular toxic goiter;

D.) diffuse toxic goiter;

19. Indicate the correct sequence of application of drugs for anaphylactic shock:

Choose one answer.

a.) prednisolone - clemastine - aminophylline - epinephrine;

B.) clemastine (tavegil) - epinephrine (adrenaline) - prednisolone - aminophylline (eufillin)

c.) epinephrine - prednisolone - clemastine - aminophylline

20. How to stop a diabetic coma?

Choose one answer.

a.) IV 40-80 ml of 40% glucose solution;

B.) IV 1 ml of 0.1% adrenaline solution

C.) IV 20 IU insulin-zinc suspension;

D.) intravenously at 0.1 U/hour short-acting insulin;

21. What is NOT an absolute contraindication to the use of estrogen preparations?

Choose one answer.

a.) uterine bleeding of unknown nature;

B.) liver disease, history of jaundice;

E.) thrombophilia;

Preview:

Subject : "Drugs affecting

to the function of the executive bodies"

Test tasks

1. Remedy for emergency treatment of a hypertensive crisis (with the manifestation or increase of signs of damage to “target organs”):

Choose one answer.

a.) methyldopa;

B.) captopril;

C.) sodium nitroprusside

D.) metoprolol;

2.AAS for the treatment of supraventricular and ventricular tachyarrhythmias:

Choose one answer.

a.) Verapamil

B.) Lidocaine

C.) Procainamide (Novocainamide)

D.) Phenytoin (Difenin)

3.AAS with minimal potential for proarrhythmic action:

Choose one answer.

a.) Propranolol (Inderal)

B.) Amiodarone

C.) Propafenone

D.) Lidocaine

4. AAS used to treat coronary artery disease:

Choose one answer.

a.) Lidocaine

B.) Verapamil

C.) Quinidine

D.) Propafenone

5.AAS, characterized by the longest half-life:

Choose one answer.

a.) Quinidine

B.) Adenosine

C.) Lidocaine

D.) Amiodarone

6. Antihypertensive neurotropic agent of peripheral action:

Choose one answer.

a.) captopril;

B.) metoprolol;

C.) nifedipine

D.) clonidine;

7. Antihypertensive agent from the group of calcium channel blockers:

Choose one answer.

a.) nifedipine

B.) metoprolol;

C.) captopril;

D.) losartan;

8. Antihypertensive agent from the group of myotropic vasodilators:

Choose one answer.

a.) captopril;

B.) diltiazem;

C.) dichlothiazide;

D.) metoprolol

9. Antihypertensive agent of central action:

Choose one answer.

a.) clonidine;

B.) pentamine

C.) sodium nitroprusside;

D.) captopril;

10. Antihypertensive agent that blocks alpha and beta adrenoreceptors:.

Choose one answer.

a.) carvedilol;

B.) metoprolol

C.) atenolol;

D.) propranolol;

11. An antihypertensive drug with a high risk of developing the effect of the first dose (severe hypotension in the orthostatic position):

Choose one answer.

a.) metoprolol;

B.) hydrochlorothiazide;

C.) captopril;

D.) prazosin

12. Antihypertensive drug contraindicated in bilateral renal artery stenosis:

Choose one answer.

a.) metoprolol;

B.) verapamil;

C.) captopril;

D.) nifedipine

13. Antihypertensive agent that reduces the formation of angiotensin II:

Choose one answer.

a.) verapamil;

B.) losartan;

C.) captopril;

D.) prazosin

14. Antihypertensive agent that reduces the level of renin in the blood:

Choose one answer.

a.) prazosin;

B.) verapamil;

C.) propranolol

D.) pentamine;

15. Antifibrinolytic action has:

Choose one answer.

a.) phytomenadione

B.) calcium chloride

C.) heparin

D.) aminocaproic acid

16. Beta-blockers are used for IHD based on:

Choose one answer.

a.) reduction in oxygen demand by reducing heart rate and myocardial contractility; b.) reducing myocardial oxygen demand by reducing preload;

C.) increase in the extraction of O2 from the blood

D.) improvement of coronary blood flow;

17. A substance that activates fibrinolysis is:

Choose one answer.

a.) Warfarin

B.) clopidogrel

C.) hirudin

D.) streptokinase

18. All cardiotonic drugs increase:

Choose one answer.

a.) atrioventricular conduction;

B.) myocardial contractility;

C.) automatism of the sinoatrial node

D.) myocardial oxygen demand;

19. The main element of the antihypertensive action of alpha-blockers:

Choose one answer.

a.) Venous vasodilation;

B.) Negative chrono- and inotropic effects

C.) Arteriolar vasodilation;

20. The main element of the antihypertensive action of beta-blockers:

Choose one answer.

a.) Negative chrono- and inotropic effects

B.) Arteriolar vasodilation;

C.) Venous vasodilation;

D.) Blockade of angiotensin II receptors;

21. For the treatment of arterial hypertension is NOT used:

Choose one answer.

a.) furosemide

B.) spironolactone;

C.) hydrochlorothiazide;

D.) mannitol;

22. For the systematic treatment of arterial hypertension, the following is NOT used:

Choose one answer.

a.) metoprolol;

B.) losartan;

C.) phentolamine

D.) nifedipine;

23. Bronchodilators from the group of sympathomimetics include:

Choose one answer.

a.) isadrin

B.) ephedrine

C.) salbutamol

24. Non-glycoside cardiotonic drugs include all drugs, EXCEPT:

Choose one answer.

a.) milrinone

B.) strophanthin (ouabaina);

C.) dopamine;

D.) dobutamine;

25. Anticoagulants of indirect action include:

Choose one answer.

a.) hirudin

B.) Sodium hydrocitrate

C.) fraxiparine

D.) warfarin

26. Cardiac glycosides (CG) include all of the following drugs, EXCEPT:

Choose one answer.

a.) digoxin;

B.) dobutamine;

C.) digitoxin;

D.) strophanthin

27. What combination of diuretics is rational?

Choose one answer.

a.) Furosemide + mannitol

B.) Mannitol + urea

C.) Dichlothiazide + triamterene

D.) Furosemide + ethacrynic acid

28. What preparations of nitroglycerin are used by patients with angina pectoris to relieve attacks?

Choose one answer.

a.) nitroglycerin in ointment

B.) nitroglycerin in sublingual tablets;

C.) nitroglycerin in microdrage (sustak);

D.) nitroglycerin in solution for intravenous administration;

29. What signs of SG overdose are life-threatening?

Choose one answer.

a.) fatigue, muscle weakness

B.) dyspeptic disorders;

C.) visual disorders;

D.) ventricular tachyarrhythmias;

30. What drugs are used to treat coronary artery disease?

Choose one answer.

a.) all of the above

B.) anti-atherosclerotic agents;

C.) antithrombotic agents;

D.) cardioprotective agents;

31. What remedy can be used to stop intestinal spasms (colic)?

Choose one answer.

a.) metamizole (analgin);

B.) metoclopramide;

C.) drotaverine (no-shpa).

D.) morphine;

E.) magnesium sulfate;

32. Which of the antacids can cause alkalosis if used systematically?

Choose one answer.

a.) magnesium trisilicate;

B.) aluminum hydroxide;

C.) calcium gluconate;

D.) sodium bicarbonate

E.) magnesium oxide;

33. What antiemetic is used for reflux, paresis of the stomach?

Choose one answer.

a.) ondansetron (zofran);

B.) chlorpromazine (chlorpromazine);

C.) metoclopramide;

D.) diphenhydramine (diphenhydramine);

E.) perphenazine hydrochloride (etaperazine)

34. What diuretic can cause hearing loss?

Choose one answer.

a.) spironolactone

B.) Dichlothiazide

C.) mannitol

D.) furosemide

35. What drug is used to enhance the contractile activity of the myometrium during childbirth?

Choose one answer.

a.) ergometrine maleate

B.) Atropine sulfate

C.) oxytocin

D.) papaverine

36. What drug is used to stop uterine bleeding?:

Choose one answer.

a.) ergometrine maleate

B.) Atropine sulfate

C.) fenoterol

D.) prostaglandin F-2a

37. Which drug belongs to direct-acting anticoagulants?

Choose one answer.

a.) fibrinolysin

B.) phytomenadione

C.) Heparin

D.) Warfarin

38. What drug is used only to prevent vomiting caused by motion sickness (seasickness)?

Choose one answer.

a.) metoclopramide (cerucal);

B.) perphenazine hydrochloride (etaperazine)

C.) diprazine (pipolphen);

D.) "Aeron";

E.) ondansetron (zofran);

39. What drug reduces myometrial contractility?

Choose one answer.

a.) fenoterol

B.) pituitrin

C.) prostaglandin F-2a

D.) Papaverine

40. The mechanism of the bronchodilator action of atropine is associated with:

Choose one answer.

a.) direct myotropic action on the smooth muscles of the bronchi

B.) blockade of m-cholinergic receptors of bronchial smooth muscles

C.) excitation of B2-adrenergic receptors

41. Mechanism of action of loop diuretics (furosemide, etc.):

Choose one answer.

a.) Increase the osmotic pressure of the fluid in the lumen of the tubules

B.) Reduce the reabsorption of sodium, chloride and potassium in the thick part of the ascending limb of the loop of Henley

c.) Increase glomerular filtration

D.) block carbanhydrase

42. Mechanism of action of thiazide diuretics?

Choose one answer.

a.) Increase the osmotic pressure of the fluid in the tubules of the nephron

B.) Increase glomerular filtration

C.) block aldosterone receptors

d.) Reduce the reabsorption of sodium, chloride in the distal tubules

43. The mechanism of expectorant action of thermopsis preparations is carried out due to:

Choose one answer.

a.) irritation of the receptors of the stomach and a reflex increase in the secretion of the bronchial glands

b.) direct stimulation of the secretion of the bronchial glands

c.) liquefaction of sputum during protein depolymerization

44. The most appropriate indication for prescribing SG is:

Choose one answer.

a.) unstable angina;

b.) CHF with severe bradycardia;

c.) CHF with multiple ventricular extrasystoles

d.) CHF with atrial fibrillation;

45. Common property of all AAS (except cardiac glycosides) used to treat tachyarrhythmias:

Choose one answer.

a.) Slow down rapid depolarization

b.) Slow repolarization

c.) Acceleration of repolarization

d.) Decreased automatism

46. ​​The main property of heparin is:

Choose one answer.

a.) Cumulative

b.) effective when taken orally

c.) Action develops after 18-24 hours

d.) Delays blood clotting "in vivo" and "in vitro"

47. Features of the action of lidocaine:

Choose one answer.

a.) Slows down rapid depolarization

b.) Accelerates repolarization

c.) Slows down conduction

d.) Raises blood pressure

48. Mark the antiplatelet agent - COX inhibitor:

Choose one answer.

a.) warfarin

b.) phytomenadione

c.) acetylsalicylic acid

d.) Sodium hydrocitrate

49. Mark the coagulant of direct action:

Choose one answer.

a.) phytomenadione

b.) thrombin

c.) aprotinin

d.) Heparin

50. Note the side effect of Eufilin:

Choose one answer.

a.) respiratory depression

b.) increased myocardial oxygen demand

c.) increase in blood pressure

51. Mark the drug related to hydrouretics:

Choose one answer.

a. 2) indapamide

b. 3) mannitol

c. 1) Dichlothiazide

d. 4) furosemide

52. Mark the drug related to saluretics:

Choose one answer.

a.) Urea

b.) Mannitol

c.) Demeclocycline

d.) Furosemide

53. Mark the drug that enhances the outflow of bile (cholekinetic):

Choose one answer.

a.) "Holenzim";

b.) dehydrocholic acid;

c.) drotaverine (no-shpa);

d.) magnesium sulfate;

e.) atropine;

f.) aminophylline (eufillin)

54. Mark the drug-cholesecretic of plant origin:

Choose one answer.

a.) Magnesium sulfate;

b.) osalmid (oxafenamide);

c.) "Allohol";

d.) Holenzim

e.) drotaverine (no-shpa);

55. Mark a laxative for emergency bowel cleansing (preparation for medical or diagnostic procedures):

Choose one answer.

a.) lactulose;

b.) magnesium sulfate;

c.) Isafenine;

d.) glycerin suppositories;

e.) phenolphthalein

56. Mark the means of substitution therapy for chronic pancreatitis:

Choose one answer.

a.) contrical;

b.) pentagastrin

c.) misoprostol;

d.) atropine;

e.) pancreatin;

57. Mark the agent that reduces the secretion of hydrochloric acid in the stomach:

Choose one answer.

a.) omeprazole;

b.) sodium bicarbonate;

c.) aluminum hydroxide

d.) histamine;

e.) pentagastrin

58. Why can coronary lytics (for example, dipyridamole) cause myocardial steal?

Choose one answer.

a.) tone the coronary vessels;

b.) increase myocardial contractility

c.) redistribute blood flow to healthy vessels to the detriment of the ischemic zone of the myocardium;

d.) expand the vessels of the systemic circulation;

59. A drug that reduces bronchial reactivity from the group of HA in bronchial asthma is used:

Choose one answer.

a.) beclamethasone dipropionate

b.) cromolyn sodium

c.) ipratropium bromide

60. The drug of choice for stopping bronchospasm is:

Choose one answer.

a.) isadrin

b.) salbutamol

c.) atropine

61. What diseases are diuretics used for planned therapy:

Choose one answer.

a.) acute poisoning

b.) Cerebral edema

c.) hypertension

d.) Pulmonary edema

62. In case of pulmonary edema, the following is used to reduce pressure in the pulmonary circulation:

Choose one answer.

a.) ganglion blockers

b.) oxygen inhalation

c.) respiratory stimulants

63. In case of pulmonary edema, inhalation of a solution of ethyl alcohol is used to:

Choose one answer.

a.) antifoam action

b.) narcotic effect

c.) dehydrating effect

64. Antiarrhythmic action of cardiac glycosides is due to:

Choose one answer.

a.) Decreased force of heart contractions

b.) Conduction retardation

c.) Decreased automatism

d.) Decreased excitability

65. An antitussive that suppresses the cough reflex and blocks the excitability of sensitive endings in the respiratory tract is:

Choose one answer.

a.) tusuprex

b.) codeine

c.) libexin

66. Mixed stimulating effect on the respiratory center has:

Choose one answer.

a.) caffeine

b.) nikethamide (cordiamin)

c.) cytiton

67. Specific undesirable side effect of angiotensin-converting enzyme inhibitors:

Choose one answer.

a.) dry cough

b.) agranulocytosis;

c.) rhinorrhea;

d.) anorexia;

68. Means for the treatment of bradyarrhythmias

Choose one answer.

a.) Verapamil

c.) Lidocaine

d.) Atropine

69. Means for the treatment of only ventricular tachyarrhythmias

Choose one answer.

a.) Propafenone

b.) Procainamide (Novocainamide)

c.) Lidocaine

d.) Verapamil

70. Means for the treatment of only supraventricular tachyarrhythmias

Choose one answer.

a.) Lidocaine

b.) Procainamide (Novocainamide)

c.) Verapamil

d.) Propafenone

71. Specify the most frequent undesirable effect of nitrates:

Choose one answer.

a.) methemoglobin formation;

b.) headache;

c.) decrease in the tone of the bile duct

d.) inhibition of platelet aggregation;

72. What is the indication for the appointment of metoclopramide?

Choose one answer.

a.) diarrhea;

b.) low acidity;

c.) hyperacidity;

d.) kinetosis (sea, air sickness);

e.) nausea, vomiting.

Preview:

Subject:"Drugs affecting

to the central nervous system"

Test tasks

1.What is true about ACK?

Choose one answer.

a.) not used for arthritis;

b.) least ulcerogenic;

c.) do not use in children under 14 years of age with fever;

d.) used as an antiplatelet agent in higher than analgesic doses

2. What is NOT a contraindication for the use of opioid analgesics?

Choose one answer.

a.) respiratory depression;

b.) myocardial infarction;

c.) traumatic brain injury

d.) acute abdominal pain of unknown origin;

3. What is used to restore breathing in case of an overdose of heroin (morphine)?

Choose one answer.

a.) naloxone;

b.) oxygen;

c.) tramadol;

d.) naltrexone

4. What is characteristic of the antipyretic effect of non-narcotic analgesics?

Choose one answer.

a.) HA cause hypothermia by suppressing heat production;

b.) appointment is obligatory at subfebrile temperature;

c.) HA reduce fever by increasing heat dissipation

d.) it is inherent in all NA in doses significantly higher than analgesic;

5. What is typical for pyrazolone derivatives (metamisole (analgin), phenylbutazone (butadione))?

Choose one answer.

a.) used as antiplatelet agents in coronary artery disease

b.) are used for long-term therapy of arthritis;

c.) are hematotoxic;

d.) no anti-inflammatory effect;

6. What is characteristic of the anti-inflammatory action of NSAIDs?

Choose one answer.

a.) improve the quality of life of patients by temporarily reducing the symptoms of arthritis;

b.) cure arthritis with a full course of treatment;

c.) inhibit all phases of inflammation;

d.) anti-inflammatory effect due to inhibition of leukotriene synthesis7. What is the cause of death in an overdose of opioid analgesics?

Choose one answer.

a.) bronchospasm;

b.) pulmonary edema;

c.) respiratory arrest;

d.) cardiac arrest

8. Extrapyramidal movement disorders - a typical unwanted side effect:

Choose one answer.

a.) Clozapine

b.) Haloperidol

c.) Olanzapine

d.) Risperidone

9. What drug is used for status epilepticus?

Choose one answer.

a.) diphenhydramine (diphenhydramine);

b.) diazepam;

c.) ethosuximide

10. What effect of opioid analgesics can be dangerous even with a single use against the background of severe pain

Choose one answer.

a.) spasms in the gastrointestinal tract;

b.) euphoria;

c.) constipation

d.) respiratory depression;

11. What effect of opioid analgesics limits their widespread use?

Choose one answer.

a.) analgesic;

b.) sedative;

c. 3) euphoric;

d. 4) spasmodic

12. Caffeine:

Choose one answer.

a.) Tones the respiratory and vasomotor centers

b.) Constricts the coronary vessels

c.) Dilates cerebral vessels

13. Moclobemide, compared with imipramine, has a stronger:

Choose one answer.

a.) Psychostimulant action

b.) Psychosedative action

c.) Alpha-adrenergic blocking action

d.) M-anticholinergic action

14. Nimesulide and celecoxib - selective COX-2 inhibitors - differ from non-selective ones (ASA, diclofenac, etc.):

Choose one answer.

a.) more efficient;

b.) less frequency of gastropathy;

c.) less allergenic;

d.) significantly lesser severity of all "PG-dependent" side effects

15. The main indication for the use of opioid (narcotic) analgesics

Choose one answer.

a.) Traumatic and visceral pain of high intensity

b.) traumatic and visceral pain of moderate intensity;

c.) neuralgia;

d.) osteoalgia;

16. Note the undesirable effects common to NA / NSAIDs (“COX- and PG-dependent”):

Choose one answer.

a.) addiction, drug dependence;

b.) heaviness, respiratory depression;

c.) allergic reactions, leukopenia

d.) gastropathy, bleeding;

17. Mark the correct statement about Ketorolac:

Choose one answer.

a.) is only effective for moderate pain;

b.) is used for long-term therapy of arthritis;

c.) not used for more than 5-7 days due to nephrotoxicity

d.) hepatotoxic;

18. In parkinsonism apply:

Choose one answer.

a.) phenytoin (difenin);

b.) carbamazepine;

c.) levodopa

19. Antiemetic effect has:

Choose one answer.

a.) Chlorpromazine

b.) Buspirone

c.) Zopiclone

d.) Diazepam

20. Anticonvulsant effect has:

Choose one answer.

a.) Haloperidol

b.) Diazepam

c.) Buspirone

d.) Chlorpromazine

21. Psychostimulant from the group of methylxanthines:

Choose one answer.

a.) Amphetamine

b.) Caffeine

c.) Moclobemide

d.) Piracetam

e.) Imipramine

22. Specific remedy for the treatment of acute poisoning with benzodiazepine derivatives of tranquilizers:

Choose one answer.

a.) Zopiclone

b.) Flumazenil

c.) Caffeine

d.) Piracetam

e.) Phenazepam

23. Tranquilizer with the longest half-life (T1 / 2 > 48 hours):

Choose one answer.

a.) Diazepam

b.) Oxazepam

c.) Lorazepam

d.) Medazepam

e.) Midazolam

24. Tricyclic antidepressant:

Choose one answer.

a.) Caffeine

b.) Amitriptyline

c.) Fluoxetine

d.) Piracetam

25. Indicate the features of the analgesic effect of NA:

Choose one answer.

a.) are effective in moderate arthralgia, myalgia, cephalgia;

b.) eliminate pain of any intensity;

c.) more effective than narcotic analgesics for severe traumatic and visceral pain;

d.) long-term use develops tolerance

26. Fluoxetine versus amitriptyline::

Choose one answer.

a.) Less toxic

b.) Has a stronger M-anticholinergic effect

c.) More clinically effective

d.) Has a stronger sedative effect

27. What is the difference between hypnotic barbituric acid derivatives and benzodiazepine derivatives?

Choose one answer.

a.) Pronounced central muscle relaxant action

b.) more disruption of the sleep structure;

c.) weak induction of microsomal liver enzymes;

28. How do partial agonists and agonists-antagonists of opioid receptors (pentazocine, buprenorphine) differ from full agonists (morphine)?

Choose one answer.

a.) stronger spasmodic action;

b.) less addictive;

c.) rectal administration is possible

d.) stronger respiratory depression;

29. Allosteric activator of GABA-A receptors:

Choose one answer.

a.) Baclofen

b.) Diazepam

c.) Buspirone

d.) Amizil

30. Antidepressant selective MAO-A inhibitor:

Choose one answer.

a.) Moclobemide

b.) Piracetam

c.) Fluoxetine

d.) Imipramine

e.) Amitriptyline

f.) Caffeine

31. Antidepressant selective serotonin reuptake inhibitor:

Choose one answer.

a.) Piracetam

b.) Fluoxetine

c.) Caffeine

d.) Imipramine

e.) Amitriptyline

32. Antimaniacal effect does not have:

Choose one answer.

a.) Haloperidol

b.) Lithium carbonate

c.) Diazepam

d.) Triftazin

33. Antipsychotic agent from the group of phenothiazine derivatives:

Choose one answer.

a.) Risperidone

b.) Olanzapine

c.) Chlorpromazine

d.) Clozapine

e.) Haloperidol

34. Atypical antipsychotic:

Choose one answer.

a.) Fluorphenazine

b.) Haloperidol

c.) Clozapine

d.) Chlorpromazine

e.) Triftazin

35. The main psychotropic effect of piracetam:

Choose one answer.

a.) Anxiolytic

b.) Sedation

c.) Mnemotropic

d.) Psychostimulant

36. For NSAIDs, all of the following drug interactions are characteristic, EXCEPT:

Choose one answer.

a.) codeine reduces the analgesic effect of NA or NSAIDs;

b.) NSAIDs weaken the effect of diuretics and a number of antihypertensive agents;

c.) aluminum-containing antacids reduce the bioavailability of NSAIDs

d.) sedatives enhance the analgesic effect of NSAIDs;

37. Day tranquilizer:

Choose one answer.

a.) Phenazepam

b.) Zopiclone

c.) Medazepam

d.) Diazepam

e.) Aminazine

38. Ventricular tachyarrhythmias potential unwanted side effect:

Choose one answer.

a.) Typical antipsychotics

b.) Tricyclic antidepressants

c.) Selective serotonin reuptake inhibitor antidepressants

d.) Benzodiazepine tranquilizers

e.) Atypical antipsychotics

39. Symptoms of acute poisoning with sleeping pills include:

Choose one answer.

a.) excitation, increase in blood pressure;

b.) coma, respiratory depression, hypoxia;

c.) temperature rise, increased reflex excitability

40. What opioid analgesics are contraindicated in myocardial infarction?

Choose one answer.

a.) pentazocine, butorphanol;

b.) morphine, promedol;

c.) fentanyl, nalbuphine

41. What drugs can be used for spasticity of skeletal muscles?

Choose one answer.

a.) strychnine, nikethamide (cordiamin), bemegride

b.) baclofen, diazepam, mydocalm;

c.) prozerin, galantamine, physostigmine;

42. Which statement regarding acetaminophen (paracetamol) is NOT true?

Choose one answer.

a.) NSAID of choice for arthritis

b.) gastrotoxic;

c.) there is no antiplatelet effect;

d.) antipyretic drug of choice for viral infections in children;

43. Which highly active opioid analgesic is preferred for pain relief in short-term painful manipulations/surgeries?

Choose one answer.

a.) morphine;

b.) pentazocine

c.) fentanyl;

d.) promedol;

44. Which drug belongs to antiepileptic drugs?

Choose one answer.

a.) sodium valproate;

b.) levodopa;

c.) cyclodol

45. Which drug belongs to sleeping pills?

Choose one answer.

a.) cyclodol;

b.) zopiclone;

c.) phenytoin (difenin);

d.) levodopa

46. ​​What drug is preferable for anesthesia of the first stage of labor?

Choose one answer.

a.) codeine

b.) metamizole (analgin);

c.) morphine;

d.) trimeperidine (promedol);

a.) copper preparations

b.) Phosphorus

c.) mercury compounds

d.) iron compounds

2. For which medicinal substance the primary pharmacological reaction is due to the influence on the processes of DNA transcription:

Choose one answer.

a.) insulin;

b.) benzylpenicillin

c.) heparin;

d.) prednisolone;

3. For which drug the primary pharmacological reaction is due to a decrease in the permeability of voltage-gated ion channels:

Choose one answer.

a.) digitoxin;

b.) lidocaine;

c.) ropine;

d.) furosemide

4. For which drug the primary pharmacological reaction is due to a decrease in the permeability of mediator-dependent (chemosensitive) ion channels:

Choose one answer.

a.) lidocaine;

b.) pipecuronium

c.) paracetamol;

d.) verapamil;

5. For which drug the primary pharmacological reaction is due to the inhibition of enzyme activity:

Choose one answer.

a.) lidocaine;

b.) adrenaline;

c.) prozerin

d.) atropine;

6. For which medicinal substance the primary pharmacological reaction is due to inhibition of the process of facilitated diffusion:

Choose one answer.

a.) adrenaline;

b.) dichlothiazide.

c.) digoxin;

d.) diazepam;

7. To reduce the concentration of poison in the blood and tissues, use:

Choose one answer.

a.) laxatives

b.) chemical antidotes

c.) adsorbents

d.) functional antidotes

8. To remove unabsorbed poison from the stomach, the latter is washed with water with the addition of:

Choose one answer.

a.) atropine solution

b.) sodium sulfate

c.) methylthioninium chloride (methylene blue)

d.) activated carbon

9. Complexones include:

Choose one answer.

a.) Pentacin

b.) naloxone

c.) sodium thiosulfate

d.) Pentamine

10. What means are used to stimulate the respiratory center:

Choose one answer.

a.) nikethamide (cordiamin); bemegrid; sulphocamphocaine;

b.) morphine; fentanyl; trimeperidine (promedol)

c.) epinephrine (adrenaline); phenylephrine (mesatone); norepinephrine (norepinephrine)

d.) drotoverin (no-shpa); metacin; papaverine;

11. Sodium thiosulfate forms low-toxic rhodanide compounds in case of poisoning:

Choose one answer.

a.) heroin

b.) cyanides

c.) atropine

d.) cardiac glycosides

12. The main goals of the treatment of acute poisoning are all except:

Choose one answer.

a.) decrease in the concentration of poison in the blood and tissues

b.) reducing further venom absorption

c.) normalization of functions of vital organs and systems

d.) slowing down the metabolism of poison

13. The functional antidote of morphine is:

Choose one answer.

a.) Diphenhydramine (Diphenhydramine)

b.) atropine

c.) naloxone

d.) bemegrid

14. A chemical antidote for an overdose of Heparin is:

Choose one answer.

a.) phytomenadione

b.) calcium chloride

c.) protamine sulfate

d.) dimercaprol (unithiol)

15. Ethyl alcohol changes the metabolism of poison in case of poisoning:

Choose one answer.

a.) methyl alcohol

b.) atropine

c.) morphine

d.) arsenic preparations


1. What is the name of the section of pharmacology that studies the absorption, distribution, biotransformation and excretion of drugs?

Pharmacokinetics.

Pharmacodynamics.

2. What is the name of the section of pharmacology that studies the types of action of drugs, pharmacological effects, mechanism of action?

Pharmacodynamics.

Pharmacokinetics.

3. The main mechanism of drug absorption in the gastrointestinal tract:

active transport.

Facilitated diffusion.

Passive diffusion across cell membranes.

Pinocytosis.

4. The main place of drug absorption is weak bases:

Small intestine.

5. The main place of drug absorption is weak acids:

Small intestine.

6. What route of drug administration provides 100% bioavailability?

Intramuscular.

Rectal.

Intravenous.

Through the mouth.

7. How will the absorption of drugs - weak acids change with a decrease in the acidity of gastric juice?

will increase.

will decrease.

8. How will the absorption of drugs - weak bases change with a decrease in the acidity of gastric juice?

will increase.

will decrease.

9. By passive diffusion, substances are easily transported through biological membranes:

Lipophilic.

Polar.

Hydrophilic.

10. Enteral route of drug administration:

Intramuscular.

Inhalation.

Sublingual.

Intravenous.

11. Parenteral route of drug administration:

Through the mouth.

Into the rectum.

Subcutaneous.

Sublingual.

12. Where does the absorption of most of the drugs take place?

In the mouth.

In the stomach

In the small intestine.

In the large intestine.

13. You can enter intravenously:

Oil solutions.

insoluble compounds.

Osmotically active compounds.

Microcrystalline suspensions.

insoluble compounds.

14. What functional changes in the body cause cardiac glycosides in heart failure?

Excitation.

Oppression.

Toning.

Calm.

15. What functional change in the body causes a drug that lowers blood pressure in arterial hypertension?

Excitation.

Oppression.

Toning.

Calm.

16. What is the name of the accumulation of a drug in the body during its repeated injections?

functional cumulation.

Sensitization.

material accumulation.

Tachyphylaxis.

17. Tolerance is:

An allergic reaction of the body to the repeated administration of the drug.

Reducing the pharmacological effect on repeated drug administration.

Irresistible urge to take the medicine again.

18. Reducing the effect of administering drugs at short intervals is:

Tachyphylaxis.

Idiosyncrasy.

Sensitization.

Addiction.

19. Side effect that may occur only with repeated administration of drugs:

Idiosyncrasy.

Teratogenic action.

mutagenic action.

Addictive.

20. Side effect that may occur only when using psychotropic drugs:

Idiosyncrasy.

Addiction.

Addictive.

Sensitization.

21. Determine the type of drug interaction: a patient with muscarine poisoning underwent gastric lavage with a suspension of activated charcoal:

Combined synergy.

chemical antagonism.

competitive antagonism.

physical antagonism.

22. Mutagenic action is:

23. Teratogenic effect is:

Damage to the genetic apparatus of the germ cell.

Violation of the differentiation of fetal tissues, causing various anomalies.

A side effect that occurs in the first 12 weeks after fertilization and causes the death of the embryo.

24. Embryotoxic action is:

Damage to the genetic apparatus of the germ cell.

Violation of the differentiation of fetal tissues, causing various anomalies.

A side effect that occurs in the first 12 weeks after fertilization and causes the death of the embryo.

CLINICAL PHARMACOLOGY

001. How the volume of distribution of fat-soluble drugs changes

in obese patients?

a) decreases

c) does not change

d) does not change or increases

d) increases

002. How does the biotransformation of drugs change on the background of smoking and drinking alcohol?

a) decreases

b) decreases or does not change

c) does not change

d) does not change or increases

d) getting stronger

003. On what main factor does the bioequivalence of a drug depend?

a) pharmacodynamic characteristics of drugs

b) physical and chemical characteristics

c) dosage form

d) manufacturing technology

e) the state of the patient's body

004. What side effects of drugs do not depend on the dose?

a) associated with the pharmacological properties of drugs

b) toxic complications,

due to absolute or relative overdose

c) secondary effects due to the violation

immunobiological properties of the body

d) immunological reactions of immediate and delayed types

e) withdrawal syndrome

005. What is the complication in newborns

can cause the introduction of magnesium sulfate to pregnant women before childbirth?

a) development of neuromuscular blockades and lethargy

b) respiratory depression

c) thrombocytopenia

d) hepatotoxic effect

e) malnutrition

006. What complications in newborns

can cause the appointment of pregnant women -adrenergic blockers?

a) hearing loss

b) damage to the skin

c) malnutrition of the placenta and fetus

d) premature closure of the ductus arteriosus

e) hemorrhagic syndrome

007. Use of which antimicrobial drugs

most safe during pregnancy?

a) aminoglycosides

b) cotrimoxazole

c) nitrofurans

d) penicillins

e) fluoroquinolones

008. Specify an antibiotic that does not have antipseudomonal activity:

a) carbenicillin

b) ampioks

c) ticarcillin

e) azlocillin

d) ceftazidime

009. A patient with pneumonia receiving antibiotic treatment,

began to complain of dizziness,

unsteadiness and staggering when walking.

What antibiotic could cause these symptoms?

a) ampicillin

b) cefoperazone

c) gentamicin

d) erythromycin

e) lincomycin

010. Specify the antibiotic that is the drug of choice

in the treatment of infections caused by staph. aureus:

a) penicillin

b) gentamicin

c) azithromycin

d) chloramphenicol

e) amoxicillin-clavulanate

011. Specify the antibiotic, which is the drug of choice

in the treatment of infections,

caused by methicillin-resistant staphylococcus aureus:

a) lincomycin

b) erythromycin

c) vancomycin

d) penicillin

e) oxacillin

012. Specify the antibiotic,

a) ampicillin

b) gentamicin

c) cefoperazone

d) metranidazole

e) tetracycline

013. Active against atypical pathogens

(mycoplasma, chlamydia, legionella):

a) gentamicin

b) erythromycin

c) ampioks

d) chloramphenicol

e) clindamycin

014. Cotrimoxazole is the drug of choice in the treatment of:

a) pneumocystis pneumonia in immunocompromised patients

b) diphtheria

c) cholangitis

d) pneumococcal pneumonia

e) amoebic dysentery

015. What antibiotic is contraindicated for patients,

receiving muscle relaxants or myasthenia gravis?

a) ampicillin

b) gentamicin

c) erythromycin

d) lincomycin

e) ciprofloxacin

016. Patient with bronchial asthma,

constantly receiving oral prednisone, teopec, berotek inhalations,

due to associated bronchopulmonary infection

erythromycin and bromhexine were prescribed.

On the third day of treatment, the patient developed headache, anxiety,

irritability, palpitations, feeling of interruption in the heart,

decrease in blood pressure, fever, nausea, vomiting.

With the toxic effect of what drug are these symptoms associated?

a) prednisone

b) teopec

c) erythromycin

d) berotek

e) bromhexine

017. The concentration of theophylline in the blood is reduced by all of these drugs,

a) phenobarbital

b) rifampicin

c) carbamazipine

d) nifedipine

e) phenytoin

018. The average therapeutic concentration of theophylline in plasma is:

a) 10-20 mcg/ml

b) 25-30 mcg/ml

c) 30-35 mcg/ml

d) 35-40 mcg/ml

e) 5-10 mcg/ml

019. With the combined action of theophylline and cimetidine, the effect of eufillin:

a) intensifies

b) increases or does not change

c) decreases

d) decreases or does not change

d) does not change

020. Duration of action of xanthines:

a) 1-2 hours

b) 2-3 hours

c) 3-4 hours

d) 6-8 hours

e) 10-12 hours

021. What is the mechanism of action of 2-adrenergic agonists?

a) inhibition of phosphodiesterase

b) inhibition of mast cell degranulation

c) blockade of histamine receptors

d) inhibition of the action of leukotrienes on the respiratory tract

e) activation of adenylate cyclase, increase in the formation of cAMP

022. Indicate the beginning, maximum action and duration of action

fenoterol (berotec):

a) immediately, 10 minutes, 6 hours

b) 15 minutes, 30 minutes, 6 hours

c) 2-3 minutes, 20 minutes, 2.5 hours

d) 5-10 minutes, 30 minutes, 6 hours

e) 30-40 seconds, 20 minutes, 3-5 hours

023. What are the most common side effects?

inhaled forms of glucocorticoids:

a) the development of osteoporosis

b) hypercortisolism

c) candidiasis of the oral cavity and pharynx

d) arterial hypertension

024. Unlike beclomethasone propionate, budesonide has:

a) greater affinity for receptors in the lungs,

undergoes active biotransformation in the liver

on first pass

b) inhibits the formation of hydrocortisone to a greater extent

c) more often leads to the development of hyperglycemia

d) more often causes an exacerbation of a broncho-pulmonary infection

e) there is no difference between the drugs

025. Specify long-acting 2-agonist:

a) salbutamol

b) terbutaline

c) fenoterol

d) orciprenaline sulfate

e) formoterol

026. Specify the drug that has the most powerful inhibitory effect

for gastric secretions:

a) omeprazole

b) cimetidine

c) famotidine

d) sucralfate

for the treatment of recurrence of duodenal ulcer:

028. A patient suffering from rheumatoid arthritis,

long time receiving NSAIDs.

What drug is indicated for this patient

to prevent ulceration?

a) sucralfate

b) gastrocepin

c) ranitidine

d) maalox

e) misoprostol

029. In the treatment of helicobacteriosis, the most effective application is:

a) ranitidine

b) oxacillin

c) de-nola

d) de-nola + ampicillin (amoxicillin)

e) maalox

030. The most rational mode of prescribing antacids

in patients with peptic ulcer:

a) 20 minutes before meals

b) immediately after eating

c) 20 minutes after eating and at night

d) one hour after eating and at night

e) regardless of food intake 4-5 times a day

031. What antihypertensive drugs

act primarily as postganglionic adrenergic blockers?

a) pentamine

b) clonidine

c) guanethidine sulfate

d) anaprilin

e) chlorthalidone

032. What antihypertensive drugs

affect the neurohumoral mechanisms of blood pressure regulation?

a) clonidine

b) captopril

c) minoxidil

d) guanethidine

e) hydrochlorothiazide

033. Specify the mechanisms of hypotensive action of veroshpiron:

a) decreased plasma renin activity

b) blockade of adrenergic receptors

c) a decrease in the volume of circulating fluid

d) decrease in total peripheral resistance

e) competitive aldosterone antagonist

034. The bioavailability of nifedipine is low due to:

a) presystemic elimination in the liver

b) low absorption

c) binding to plasma proteins

d) inactivation in the gastrointestinal tract

035. Specify the duration of the hypotensive action of a single dose of clonidine

when taken orally:

a) 1-2 hours

b) 6-8 hours

c) 10-12 hours

d) 2-24 hours

e) up to 3 days

036. Relief of uncomplicated hypertensive crisis should be started:

a) with intramuscular injection of 1.0 ml of 0.01% clonidine solution

b) with 40 mg of furosemide orally

c) with 10-20 mg of nifedipine sublingually

d) with 40 mg of anaprilin inside

e) with intravenous 1.0 ml of 0.5% phentolamine solution

037. Captopril is contraindicated in patients:

a) cirrhosis of the liver

b) with chronic renal failure

c) with cor pulmonale

d) with diabetes

e) with peptic ulcer

038. Specify the mechanism of action of nitroglycerin:

a) blockade of -adrenergic receptors

b) antispasmodic, myotropic action

on the smooth muscles of the vascular wall

c) blockade of small calcium channels of the cell membrane

d) increased activity of α-receptors

vascular wall of the coronary arteries

e) increases the slow flow of calcium into the cell

039. What are the contraindications to prescribing nitrates?

a) acute myocardial infarction

b) arterial hypertension

c) hypotension

d) bradycardia

e) atrioventricular blockade

040. What causes the antianginal effect of -adrenergic blockers?

a) expansion of the coronary vessels

b) decrease in post- and preload on the heart

c) decreased work of the heart

d) central mechanism of action

e) increased myocardial oxygen demand

041. Specify a -blocker of selective action:

a) scolded

b) trazikor

c) whiskey

d) sector

042. What is the bioavailability of propranolol when taken orally?

043. What is the bioavailability of verapamil when taken orally?

044. Specify contraindications to the appointment of nifedipine:

a) arterial hypertension

b) heart failure

c) bronchial asthma

d) arterial hypotension

e) atrioventricular blockade of the 2nd degree

045. Specify the drug belonging to the 1st group of antiarrhythmic drugs

(membrane stabilizing action):

a) lidocaine

b) isoptin

c) cordarone

d) quinidine

e) difinin

046. Specify the drug belonging to the 2nd group of antiarrhythmic drugs

(local anesthetics):

a) mexitil

b) obzidan

c) whiskey

d) cordarone

e) novocainamide

047. Specify the drug belonging to the 3rd group of antiarrhythmic drugs

(-blockers):

a) lidocaine

b) trazikor

c) cordarone

d) quinidine

e) isoptin

048. Specify the duration of action of lidocaine:

a) 20 minutes

b) 60 minutes

c) 1.5-2 hours

e) 12 hours

049. Specify the half-life of cordarone:

a) 4-6 hours

b) 1-2 hours

c) 20-24 hours

050. Specify the time to reach the maximum concentration of isoptin

in plasma when taken orally:

a) 10 minutes

b) 50 minutes

c) 1.5-2 hours

d) 8 hours

e) 10 hours

051. Specify the local anesthetic,

with the most pronounced antiarrhythmic effect:

a) lidocaine

b) trimekain

c) xicaine

d) novocaine

e) mexityl

052. The development of tolerance to nitrates depends mainly on:

a) from the route of administration of nitrate

b) from the time of reaching the maximum concentration in the blood

c) duration of action

d) combination with other drugs

d) none of the above factors

does not affect the development of tolerance

053. Unlike isosorbide dinitrate, 5-isosorbide mononitrate:

a) does not undergo primary presystemic elimination

when passing through the liver

b) does not cause the development of tolerance

c) does not cause methemoglobinia

d) does not cause headaches

e) there are no differences between drugs

054. Name an anti-inflammatory agent of prolonged action:

a) aspirin

b) analgin

c) piroxicam

d) indomethacin

e) orthophene

055. When taking salicylates, their lowest concentration is observed:

a) in the kidneys

b) in the liver

c) in the myocardium

d) in the lungs

d) in the brain

056. Name glucocorticoid preparations with prolonged action:

a) prednisone

b) polcortolon

c) dexamethasone

d) Kenalog

e) methylprednisolone

057. Specify the side effect of nicotinic acid:

a) lipodystrophy

b) hyperuricemia

c) rhabdomyosis

d) visual impairment

e) bronchospasm

058. Specify the side effect of bile acid sequestrants:

a) itchy skin

b) diarrhea

c) constipation

d) visual impairment

e) depression

059. What antihistamine drug is contraindicated

a) suprastin

b) pipolfen

c) diphenhydramine

d) tavegil

e) fenkarol

060. What antihistamine drug is contraindicated

in the treatment of allergic reactions to the administration of drugs,

a) pipolfen

b) suprastin

c) diphenhydramine

d) tavegil

e) fenkarol

061. The half-life of drugs is:

a) the time to reach the maximum concentration of the drug in plasma

b) the time during which the drug reaches the systemic circulation

c) the time during which the drug is distributed in the body

d) the time for which the concentration of the drug in plasma decreases by 50%

e) the time it takes for half of the administered dose to reach the target organ

062. Therapeutic index is:

a) the therapeutic dose of the drug

b) the ratio of drug concentration in an organ or tissue

to its plasma concentration

c) the range between the minimum and maximum

d) percentage of non-protein-bound drug

e) range between minimum and maximum

therapeutic drug concentrations

063. Competitive receptor drugs include:

a) non-steroidal anti-inflammatory drugs

b) -blockers

c) loop diuretics

d) nitrates

e) fluoroquinolones

064. When prescribing the following medicines

the function of both the liver and kidneys should be taken into account:

a) lipophilic, forming inactive metabolites

b) lipophilic, forming active metabolites

c) hydrophilic

d) hepatotoxic

e) nephrotoxic

065. The selectivity of the action of a medicinal substance depends on:

a) from the half-life

b) on the method of reception

c) from the connection with the protein

d) on the volume of distribution

e) on the dose

066. The following groups of side effects are strictly dose-dependent:

a) pharmaceutical

b) toxic

c) allergic

d) mutagenic

e) withdrawal syndrome

067. List the groups of drugs with a narrow therapeutic index:

a) blockers

b) penicillins

c) cardiac glycosides

d) methylxanthines

e) powerful diuretics

068. The drug of choice in the presence of atypical pathogens

(mycoplasma, chlamydia) is:

a) erythromycin

b) metronidazole

c) gentamicin

d) carbenicillin

e) cefuroxime

069. Drugs of choice in the presence of atypical pathogens

(mycoplasma, chlamydia) are:

a) macrolides

b) penicillins

c) aminoglycosides

d) cephalosporins

e) sulfonamides

070. Specify an antibacterial drug,

having the highest antianaerobic activity:

a) erythromycin

b) ampicillin

c) tetracycline

d) gentamicin

e) cefotetan

071. Intestinal dysbacteriosis

Cause all of the following antibiotics except:

a) semi-synthetic drugs

b) tetracyclines

c) fluoroquinolones

d) oral cephalosporins

e) macrolides

072. The following antibacterial drugs are nephrotoxic,

a) gentamicin

b) carbenicillin

c) erythromycin

d) cefazolin

e) vancomycin

073. Specify the antibacterial drug,

not active against pneumococcus:

a) azithromycin

b) penicillin

c) ceftriaxone

d) ciprofloxacin

e) chloramphenicol

074. Choose a combination of antibacterial drugs,

with synergy of action and safety:

a) penicillins + tetracyclines

b) penicillins + cephalosporins

c) penicillins + macrolides

d) penicillins + aminoglycosides

e) penicillins + sulfonamides

075. Good penetration through the blood-brain barrier

the following antibacterial drugs:

a) penicillins

b) macrolides

c) tetracyclines

d) aminoglycosides

e) cephalosporins

076. The drug of choice for lobar pneumonia is:

a) cefaclor

b) doxycycline

c) methicillin

d) cefotaxime

e) penicillin

077. The drug of choice for pharyngitis is:

a) cefaclor

b) tetracycline

c) ceftazidime

d) ofloxacin

e) penicillin

078. New generation of macrolide antibiotics

has the following advantages:

a) high bioavailability

b) a wide range of antibacterial action

c) bactericidal action

d) long half-life

e) renal route of excretion

079. Fluoroquinolones differ from quinolones in the following properties:

a) a wide antibacterial spectrum of action

b) bacteriostatic action

c) high penetration into the tissue

d) post-antibacterial effect

e) oral route of administration

080. Choose a drug,

maximally suppressing the secretion of hydrochloric acid:

a) pirenzepine

b) cimetidine

c) carbenoxolone

d) antacids

e) omeprazole

081. The maximum number of side effects among H2-blockers

calls:

a) cimetidine

b) roxatidine

c) nizatidine

d) ranitidine

e) famotidine

082. Synthetic analogues of prostaglandins (enprostil, misoprostol)

produce the following effects:

a) antisecretory action

b) secretion of barbiturates

c) mucus formation

d) reparative action

083. Inhibits the metabolism of other drugs:

a) omeprazole

b) carbenoxolone

c) cimetidine

d) famotidine

e) gastrocepin

084. The duration of the antisecretory action of omeprazole is:

a) 2-4 hours

b) 8-10 hours

c) 16-20 hours

e) 3 days

085. Specify the drug,

which is a powerful stimulant of mucus formation in the stomach:

a) carbenoxolone

b) platifillin

d) omeprazole

e) metoclopramide

086. In the presence of renal failure

dose adjustment is required:

a) prostaglandin analogs

b) omeprazole

c) H2 blockers

d) sucralfate

e) anticholinergics

087. Specify the antisecretory drug blocking the "proton pump"

a) metoclopramide

b) carbenoxolone

c) pirenzepine

d) sucralfate

e) omeprazole

088. In case of kidney pathology

the following changes in the pharmacogenetics of drugs occur:

a) impaired renal excretion

b) an increase in the concentration of drugs in the blood plasma

c) decreased plasma protein binding

d) an increase in the half-life

e) decrease in bioavailability

089. Cirrhosis of the liver causes the following changes in the pharmacokinetics of drugs:

a) decrease in first pass metabolism

b) decreased plasma protein binding

c) an increase in the half-life

d) increase in bioavailability

e) decrease in the volume of distribution

090. In case of heart failure

the following changes in the pharmacokinetics of digoxin are observed:

a) decrease in absorption in the gastrointestinal tract by 30%

b) decreased plasma protein binding

c) increased metabolism in the liver

d) decreased renal excretion

e) an increase in the half-life

091. Alcohol causes:

a) to increase the absorption of drugs

c) to slow down metabolism in the liver

d) decrease in renal excretion

e) to an increase in the half-life

092. Nicotine leads to:

a) to reduce the absorption of drugs

b) to increase the volume of drug distribution

c) to increase the connection with plasma proteins

d) to accelerate metabolism in the liver

e) to increase renal excretion of drugs

093. In case of angina pectoris, the following is indicated:

a) nifedipine

b) propranolol

c) capoten

d) enalapril

e) clonidine

094. In case of Prinzmetal's angina (vasospastic) it is indicated:

a) nifedipine

b) obzidan

c) dipyridamole

d) dopegit

e) captopril

095. The criterion for the effectiveness of an antianginal agent is:

a) increase in load time on the PEM > 1 minute

b) increase in the amount of consumed NTG

c) VEM time increase - samples >2 minutes

d) reduction of load time

e) the transition of the patient from the 2nd to the 3rd functional class of angina pectoris

096. Antianginal drugs include:

a) chimes

b) capoten

c) aspirin

d) verapamil

097. To stop an attack of angina pectoris, the following are used:

a) joint

b) nitrong

c) nitroglycerin

d) verapamil

e) diltiazem

098. From antianginal drugs

with a combination of coronary artery disease and arterial hypertension, it is indicated:

a) joint

b) verapamil

c) captoprilil

d) chimes

e) enalapril

099. The development of tolerance is most likely when using:

a) trinitrolonga

b) sustaka

c) sublingual nitroglycerin

d) isosorbitol-5-mononitrate

e) nitrong

100. The method of monitoring the effectiveness of antianginal therapy is:

a) Holter ECG monitoring

b) control of blood lipid levels

c) daily monitoring of blood pressure

d) measurement of respiratory function (functions of external respiration)

e) measurement of blood pressure in ortho- and clinostasis

101. The drug of choice for angina pectoris in a patient with bradycardia is:

a) pindolol

b) propranolol

c) verapamil

d) diltiazem

e) metoprolol

102. Drug of choice for angina pectoris

in a patient with heart failure is:

a) verapamil

b) corinfar

c) diltiazem

d) acebutalol

e) nitrosorbitol

103. Inhaled corticosteroids include:

a) hydrocortisone

b) beclomethasone

c) prednisone

d) polcortolon

e) dexamethasone

104. Long-acting selective 2-agonists include:

a) fluticasone

b) salmeterol

c) salbutamol

d) fenaterol

e) terbutaline

105. To stop an attack of bronchial asthma, the following is used:

a) ipratropium bromide

b) teopec

d) salbutamol

e) zaditen

106. Membrane stabilizer for oral administration is:

a) ketotifen

b) nedocramil sodium

c) sodium chromoglycate

d) suprastin

e) ipratropium bromide

107. In the syndrome of "locking" apply:

a) salbutamol

b) fenoterol

c) teopec

d) eufillin

e) adrenaline

108. Mucolytic drugs include:

a) codeine

b) sodium chromoglycate

c) acetylcysteine

d) salmeterol

e) theophylline

109. With simultaneous use

increases the concentration of theophylline in the blood:

a) ofloxacin

b) penicillins

c) ceftriaxone

d) gentamicin

e) biseptol

110. With simultaneous use

reduces the concentration of theophylline in the blood:

a) pefloxacin

b) cimetidine

c) rifampicin

d) erythromycin

e) ampioks

111. With increased pressure in the pulmonary artery

in a patient with bronchial asthma:

a) verapamil

b) nifedipine

c) digoxin

e) beclomethasone

112. In case of bronchial asthma against the background of chronic bronchitis, the following is indicated:

a) ipratropium bromide

b) adrenaline

c) ephedrine

d) ketotifen

e) suprastin

113. Specify the preferred route of drug administration

in congestive heart failure:

a) rectal

b) sublingual

c) inside

d) intravenous

e) skin

114. List drugs,

having a direct positive inotropic effect:

a) digoxin

b) dopamine

c) norepinephrine

d) eufillin

e) hydralazine

115. Specify the states,

increasing sensitivity to cardiac glycosides:

a) advanced age

b) thyrotoxicosis

c) cor pulmonale

d) hypokalemia

e) congestive heart failure

116. List the drugs, when interacting with which

The concentration of digoxin in the blood may increase:

a) phospholagel

b) quinidine

c) verapamil

d) amiodarone

117. List the factors slowing down the absorption of cardiac glycosides

from the gastrointestinal tract:

a) chronic renal failure

b) congestive heart failure

c) stomach ulcer

d) co-administration with antacids

118. List the factors

providing the greatest safety and effectiveness of diuretics

in long-term treatment of heart failure:

a) maximum doses

b) average doses

c) minimum doses

d) daily intake

e) intermittent reception

119. Specify the most effective diuretic

for the treatment of congestive heart failure

with the development of secondary hyperaldosteronism:

a) ethacrynic acid (uregide)

b) chlorthalidone (hygroton)

c) acetazolamide (diacarb)

d) spironolactone (veroshpiron)

e) triampur

120. Specify the main therapeutic effect of nitrosorbide

in patients with heart failure:

a) expansion of predominantly arterioles and a decrease in afterload

b) expansion of predominantly venules and a decrease in preload

c) direct positive inotropic effect

d) increased diuresis and decreased preload

121. List drugs that have anticholinergic side effects:

a) lidocaine

b) quinidine

c) amiodarone (cordarone)

d) verapamil

e) procainamide (novocainamide)

122. List the groups of antiarrhythmic drugs,

having an antifibrillatory effect:

a) cardiac glycosides

b) calcium antagonists (group 4)

c) blockers (group 2)

d) amiodarone, bretylium tosylate (group 3)

e) quinidine, procainamide and other drugs of group 1a

123. List drugs,

that can provoke an attack of atrial tachyarrhythmia

with Wolff-Parkinson-White syndrome:

a) digoxin

b) amiodarone

c) verapamil

d) propranolol

e) ethmozine

124. Specify drugs, which are characterized by arrhythmogenic effect:

a) aymalin

b) mexiletine

c) propafenone

d) amiodarone

125. List the indications for treatment with antiarrhythmic drugs:

a) arrhythmias causing hemodynamic disturbances

b) subjective arrhythmia intolerance

c) violation of the rhythm of high gradations

d) frequent rhythm disturbances

126. List drugs,

the main antiarrhythmic action of which

associated with prolongation of atrioventricular conduction:

a) propranolol

b) lidocaine

c) verapamil

d) digoxin

e) procainamide

127. Specify the effects of the interaction of amiodarone and disopyramides

(rhythmylen, norpace):

a) slowing down the metabolism of disopyramide

b) slowing down the metabolism of amiodarone

c) increased risk of side effects of disopyramide

d) increased risk of side effects of amiodarone

128. Specify the side effect of nifidepine:

a) bradycardia

b) bronchospasm

c) swelling of the legs and feet

d) development of F-V blockade

e) ulcerogenicity

129. Specify the drug,

the mechanism of its hypotensive action

is a blockade of -receptors:

a) clonidine

b) prazosin

c) propranolol

d) capoten

e) verapamil

130. The drug of choice

in a patient with hypertension and congestive circulatory failure

is:

a) enalapril

b) nitrosorbide

c) clonidine

d) adelfan

e) pentamine

131. The optimal drug for long-term antihypertensive therapy should:

a) affect metabolism

b) have ricochet reactions

c) have a withdrawal syndrome

d) have a stable concentration in the blood

e) cause orthostatic reactions

132. List the groups of antihypertensive drugs,

reducing the activity of the renin-angiotensin-aldosterone system:

a) ACE inhibitors

b) blockers

c) central -agonists

d) thiazide diuretics

e) calcium antagonists

133. Specify antihypertensive drugs,

which must be used with caution

with a combination of diabetes mellitus and hypertension:

a) verapamil

b) propranolol

c) diltiazem

d) hypothiazide

e) enalapril

134. Carrying out drug monitoring

required in the treatment of the following groups of drugs:

a) anticonvulsant

b) 2-sympathomimetics

c) methylxanthines

d) glucocorticoids

e) M-cholinomimetics

135. The development of asystole is possible with a combination of propranolol:

a) with phenobarbital

b) with furosemide

c) with verapamil

d) with phenitine

e) with cimetidine

136. The risk of toxic effects increases with the combination of gentamicin:

a) with furosemide

b) with penicillin

c) with methylxanthines

d) with macrolides

e) with glucocorticoids

CLINICAL PHARMACOLOGY

114 - a, b, c, d, e

115 - a, b, c, d, e

088 - a, b, c, d

089 - a, b, c, d

090 - a, b, d, e

124 - a, b, c, d, e

078 - a, b, c, d

079 - a, c, d, e

083 - a, b, c, d

Choose the correct statement: a) bioavailability is the amount of drugs entering the systemic circulation, expressed as a percentage of the administered dose, b) bioavailability is determined by the amount of drug adsorption in the gastrointestinal tract and the severity of the effect of the first pass through the liver. c) bioavailability is determined by the formula: F = AUC (in / m or inside) / AUC (in / in). d) the bioavailability of drugs when administered intramuscularly is determined by the degree of its absorption and biotransformation in the body.
Answer: a B C

2.
Answer: Atrovent

3.

Answer: a, d

4.

Answer:

5.
Answer:

6.

e) Xylitol
Answer: a, c

7.

Answer: a,b,d

8.
Answer:

9. was admitted with an acute myocardial infarction that occurred 5 hours ago. Appointments: anaprilin 20 mg 4 times a day orally, heparin intravenously at 10,000 units every 4 hours. At the same time, it was possible to achieve an increase in blood clotting time up to 18-23 minutes. The next day, the patient was diagnosed with right-sided lower lobe pneumonia. Benzylpenicillin sodium salt (1,000,000 IU every 4 hours) was prescribed intravenously. After 4 hours, the blood clotting time was 8 minutes. What is your tactic?
Answer:

10.

11.
Answer: Vit.B12 at a dose of 500 mcg/day every other day, folic acid at a dose of 1.5 mg/day, ferrous sulfate (80 mg Fe2+) once a day

12.

Answer: Vit.C

13.

Answer: Cerebrolysin

14.
allergies (to butadion, heparin, metindol, penicillin, theophylline). In the hospital, reopyrin was prescribed 5 ml intramuscularly 1 time per day, hydrocortisone hemisuccinate 100 mg into the cavity of the knee joints, tavegil 0.001 g 2 times a day. per day. After 3 days, b-noy developed itchy erythematous rashes on the skin of the trunk. What is the most likely cause
deterioration?
Answer:

15.



Answer: a,b,e,f,h,i

16.
Answer: After a few months

17.


Answer: a, b, c, e, f

18.
Answer: a, b, c, d, e, g, h

19.
Answer:

20.
Answer: Ciprofloxacin

21. The phenomenon of the first passage of drugs through the liver depends on: a) blood supply to the liver, b) drug binding to protein, c) activity of hepatocyte enzymes, d) level of drug excretion, e) absorption rate
Answer: a, in

22. Drugs affecting microsomal liver enzymes: inducers of microsomal liver enzymes: a) penicillin, b) nitroglycerin, c) phenobarbital, d) furosemide, e) butadione, f) cortisol, g) propranolol, h) cimetidine, i) levomycetin, k ) diphenin
Answer: c,d

23. was admitted to the department with a woman for pain in the right mammary gland, an increase in T. to 39.5 C. She fell ill 3 days ago, on the 10th day after birth. Upon admission to the department in the upper outer quadrant of the right mammary gland, hyperemia of the skin, a massive infiltrate with a fluctuation in the center were found. Diagnosis: acute right-sided mastitis. Wound culture taken. Determine antibiotic of first choice.
Answer: Cefazolin

24.

Answer: Anaphylactic reaction

25.

Answer: Levomycetin

26.
Answer:

27.
Answer: biguanides

28.

Answer: Hypotension, dizziness.

29.

Assess the doctor's actions.

30. Patient D., 53 years old, diagnosed with coronary artery disease, stable angina??? FC, postinfarction cardiosclerosis, atrial fibrillation, HNK?? B Art. He took strophanthin, digoxin, furosemide, panangin in average therapeutic doses. Unexpectedly, the patient's temperature rose to 38.4°C, cough, shortness of breath, crepitus in the lungs on the right appeared. On the radiograph of the lungs on the right in the lower lobe, an area of ​​infiltration is determined. Gentamicin, sulfocamphocaine, suprastin were added to the treatment.

Answer:

31.

Answer: Phentolamine.

32.

Answer: d,d

33.

Answer:

34.
Answer:

35.
Answer: Enalapril.

36.

Answer: a,b,d

37.

Answer:

38.
Answer:

39.
Answer:

40.

Answer: all of the above

41. Drugs affecting microsomal liver enzymes: inhibitors of microsomal liver enzymes:
a) penicillin, b) nitroglycerin, c) phenobarbital,
d) furosemide, e) butadione, f) cortisol, g) propranolol,
h) cimetidine, i) chloramphenicol, j) diphenin
Answer: h,i

42.
Answer: After 7-14 days

43. Specify a combination of drugs that leads to competition for protein binding, which can lead to a dangerous increase in the content of the free fraction of one of the drugs in the blood and the appearance of symptoms of its overdose:
Answer: neodicumarin and butadione

44. Choose drugs with a narrow therapeutic window:
a) penicillins, b) anticonvulsants,
c) antiarrhythmic drugs, d) digoxin, e) methotrexate, f) theophylline, g) cyclosporine, h) macrolides
Answer: b, c, d, e, f, g

45. Specify combinations of drugs in which, due to competition for protein binding, an increase in the concentration of the free fraction of one of them in the blood plasma occurs: a. strophanthin and miscleron, b. digitoxin and miscleron, c. neodicumarin and butadione, d. nifedipine and hydrochlorothiazide
Answer: b,c

46. frequent ventricular extrasystoles and paroxysms of atrial fibrillation were detected. HR 74 per minute, blood pressure 140/80 mm Hg. -nogo: a) Quinidine,
b) Bonnecor, c) Etatsizin,

Answer: a, b

47. It is known that with a combination of quinidine and digoxin, glycoside intoxication is often observed. What is it associated with? Pharmacodynamic interaction:
Answer: synergy

48. It is known that with a combination of quinidine and digoxin, glycoside intoxication is observed. What is it associated with? Pharmacokinetic interaction, the effect of quinidine on:
Answer: protein bond

49. Critical periods of intrauterine development:
A. period of pre-implantation development (1 week)
b. the stage of embryogenesis ends by 8 weeks.
V. the stage of embryogenesis ends by 8 months.
the period immediately before childbirth
Answer: a,b,d

50. Choose from the following drugs that have the following properties: Antimicrobials, the use of which is practically safe during pregnancy: a. sulfonamides, including biseptol,
b. aminoglycosides, tetracyclines, rifampicins, metronidazole (in the 1st trimester of pregnancy), c. penicillins, cephalosporins, erythromycin, lincomycin, fusidine, d. antimycotic agents, antineoplastic
antibiotics.
Answer: V

51. Metronidazole was prescribed to a nursing mother, indicate the side effects:
a. increased excitability, tachycardia, b. appetite suppression, vomiting, c. CNS depression, respiration, weight loss, d. increased prolactin secretion, breast engorgement, e. adrenal hypoplasia, metabolic disorders, increased risk of bilirubin encephalopathy, e. hemorrhages, respiratory failure, acidosis, hemopoiesis suppression, anemia, malnutrition, dysbiosis.
Answer: b

52. Antimicrobials of the first choice in newborns: a. benzylpenicillin, oxacillin, carbenicillin, gentamicin, amikacin, b. benzylpenicillin, oxacillin, bicillins, cefazolin, cefotaxime, erythromycin, lincomycin, nystatin, c. carbenicillin, gentamicin, sisomycin, amikacin, tobramycin, tseporin (with the ineffectiveness of first-generation cephalosporins), g. erythromycin, lincomycin, nystatin, levorin, carbenicillin,
gentamicin, sisomycin
Answer: b

53.
Answer:

54. The main features of the pharmacokinetics of drugs in the elderly:
a. decrease in the rate of absorption, b. acceleration of absorption, c. decrease in the rate of distribution, d. acceleration of distribution, e. decrease in the binding of drugs to plasma proteins, e. increase in the binding of drugs to plasma proteins, g. slowdown in metabolism, h. acceleration of metabolism,
and. slowing down the excretion of drugs, to. accelerating the excretion of drugs.
Answer: a, c, e, g, i

55.
Answer: b, c, d

56. Specify the side effects of beta-blockers: a) bradycardia, b) arterial hypotension, c) bronchospasm, d) tachycardia, e) dysfunction of the thyroid gland,
f) intermittent claudication, g) AV blockade
Answer: a, b, c, f, g

57.
conditions: a) Natural course of the disease, b) Development of tolerance to nitrates, c) Intercoronary steal syndrome, d) Occurrence of rebound syndrome e) Idiosyncrasy phenomena
Answer: a, b

58. Specify the side effects of amiodarone: a) bradycardia, b) arterial hypotension, c) bronchospasm, d) tachycardia, e) dysfunction of the thyroid gland, f) intermittent claudication, g) AV blockade
Answer: a, c, e, g

59. How will your anti-anginal therapy change if a cerebral stroke occurs during nitrate therapy?
Answer: the abolition of nitrates and the appointment of an antianginal drug of another group

60. Which antihypertensive drugs are considered the safest for elderly patients: a) beta-blockers, b) ganglion blockers, c) sympatholytics, d) slow calcium channel blockers, e) thiazide
diuretics, e) ACE inhibitors.
Answer: d,d

61. Cordaron treatment regimen:
Answer: according to the scheme, which involves a gradual decrease in the dose from 600 mg to 200 mg per day

62. How do MAO inhibitors (antidepressants) affect the pressor effect of direct and indirect adrenostimulants?
Answer: enhance the effect

63. Non-benzodiazepine" benzodiazepine receptor agonist:
Answer: Zolpidem

64. Hypnotic - a compound of the aliphatic series:
Answer: Chloral hydrate

65.

Answer: A (b)

66. Protamine sulfate is prescribed for overdose:
Answer: Heparin

67. What method of detoxification is most effective in case of poisoning with substances that bind to blood proteins and lipids?
Answer: Hemosorption

68. The principle of action of naloxone in acute morphine poisoning:
Answer: Interferes with the action of morphine on opioid receptors

69. Specify the drugs with antioxidant properties: a) verapamil b) vit.A, c) vit.K, d) vit.C, e) vit.E, f) selenium, g) carnosine, h) doxycycline
Answer: b, d, e, f, g

70. What are the effects of neuroleptics?
a) antipsychotic b) sedative c) antiemetic
Answer: a B C

71. A 64-year-old patient developed an acute attack of angle-closure glaucoma with severe pain in the right eye radiating to the head. Nausea and vomiting, shortness of breath appeared, signs of a type 2 hypertensive crisis with a heart rate of 62 were revealed.
in min. AP 200/140 mmHg. B-Naya has been suffering from hypertension for many years. There are a large number of moist fine bubbling rales in the lungs. A. Clopamid, b. Veroshpiron, c. Hypothiazid, g. Furosemide IV, D. Diakarb:
Answer: d,d

72. has been suffering from diabetes mellitus for 15 years, for which he receives insulin at 70 units / day, which maintains the level of glycemia within 7.5-8.6 mmol / l. Recently, blood pressure has begun to rise to
170/90-180/100 mm Hg in connection with which the attending physician prescribed obzidan at a daily dose of 120 mg. What side effects should be expected with this combination of drugs? a. Hyperglycemia up to coma b. Heart failure c. Hypoglycemia up to coma d. Orthostatic hypotension e. Hypertension
Answer: b,c

73. , suffering from hormone-dependent bronchial asthma, was prescribed prednisone (5 mg daily), salbutamol (inhalation of 2 doses of an aerosol 4 times a day). b-th exacerbation of bronchial asthma developed. What is the reason for this?
A. Phenobarbital accelerated the biotransformation of: a. salbutamol, b. prednisolone, B. Phenobarbital accelerated the excretion of: a. salbutamol, b. prednisolone, B. Phenobarbital slowed down the excretion of: a. salbutamol, b. salbutamol, b.prednisolone
Answer: A (b)

74. suffers from coronary artery disease, angina pectoris III FC. Heart rate 90 per minute, blood pressure 150/80 mm Hg. History of chronic bronchitis with bronchospastic syndrome in remission. for antianginal therapy. a. Nitrates and verapamil, b. Nitrates and atenolol
c) Nitrates and anaprilin, d) Nitrates and nifedipine,
e) Nifedipine and amiodarone
Answer: A

75. for angina, he takes nitrosorbide 10 mg 4 times a day. Heart rate 80 in mi.BP 140 / 80 mm Hg. After 1 month after the start of therapy, angina pectoris attacks became more frequent. What are the possible causes of deterioration
conditions: a) Natural course of the disease, b) Development of tolerance to nitrates, c) Intercoronary steal syndrome, d) Occurrence of rebound syndrome, e) phenomena of idiosyncrasy
Answer: a, b

76. attacks of angina pectoris are noted with moderate physical exertion. In the anamnesis, a collaptoid state after a single dose of sublingual nitroglycerin (since then, b-noy nitroglycerin has not been taken). Concomitant diseases-GB (working level of blood pressure 160/100 mm Hg.
Art., hypofunction of the thyroid gland. At the time of examination, blood pressure was 190/100 mm Hg, heart rate was 72 per minute. The patient is contraindicated:
Answer: Amiodarone

77. due to arterial hypertension of the 2nd degree receives 0.000075 g of clonidine 4 times a day. In connection with the development of senile depression, melipramine was prescribed. 3 days after the appointment of melipramine, a hypertensive crisis occurred. conditions: a) Consequence of the natural course of the disease, b) Consequence of the hypertensive effect of melipramine, c) Consequence of adverse drug interactions, d) Consequence of possible discontinuation of the drug intake and development of the withdrawal syndrome.
Answer: b, c, d

78. due to the hypertensive crisis, sodium nitroprusside was administered intravenously in large doses (at a rate of 8 μg/min). Shortness of breath, acrocyanosis, pressing pains behind the sternum, muscle twitches appeared. What is the reason for the worsening of the b-th condition?
Answer: Toxic effect of cyanides

79. frequent ventricular extrasystoles and paroxysms of atrial fibrillation were detected. HR 74 per minute, blood pressure 140/80 mm Hg.
for further treatment of b-nogo: a) Quinidine, b) Bonnecor, c) Etatsizin,
d) Mexitil, e) Verapamil, f) Propranolol
Answer: a, b

80. paroxysmal supraventricular tachycardia against the background of WPW syndrome. Aymalin was chosen to stop the attack. Determine the optimal treatment regimen with the selected drug: a) 1 mg/kg IV for more than 10 minutes, if necessary, repeat after 30 minutes, b) 50 mg IV in leak-
3-5 minutes in 10 ml of 5% glucose solution or isotonic NaCl solution or / m, c) 0.5-1 g / in every 2 minutes, 0.1-0.2 g is administered or / m
d) after parenteral administration, prescribe orally 100 mg 4-5 times a day, maintenance dose of 50 mg 3-4 times a day
Answer: a, d

81. A 28-year-old with a diagnosis of SLE against the background of CRF developed edema of the legs, an enlarged liver. An echocardiographic study determined a decrease in cardiac output. Heart rate 95 per minute, blood pressure 170/100 mm Hg. What cardiac glycosides are indicated for the patient?
Answer: Digitoxin

82. A 28-year-old with a diagnosis of SLE against the background of CRF developed edema of the legs, enlarged liver. Echocardiography revealed a decrease in cardiac output. Heart rate 95 per minute, blood pressure 170/100 mm Hg The patient takes digitoxin. In connection with the appearance of a convulsive syndrome, phenobarbital (0.3 g / day) was additionally prescribed.
Answer: After 7-14 days

83. 57 years old for post-infarction atherocardiosclerosis, congestive heart failure of the 2nd degree receives 40 mg of furosemide IV and 300 mg
veroshpiron inside. What diuretic therapy will you prescribe to b-nom in case of refractoriness?
Answer: Furosemide 80 mg IV and spironolactone 300 mg orally

84. suffers from non-atopic bronchial asthma, accompanied by profuse bronchorrhea. Pulse 62 min. BP 140/80 mm Hg. Which drugs are more preferable?
Answer: Atrovent

85. stubbornly recurrent bronchial obstruction syndrome with reduced sensitivity to cholinergic and adrenotropic drugs. Bronchial asthma has been suffering for more than 10 years. What can be prescribed to reduce the frequency and severity of asthma attacks: a) Beta inhalation
2-adrenergic stimulants more than 6 times a day, b) Inhalation of an m-anticholinergic blocker, c) Administration of adrenaline s / c in a larger dose than usual to relieve bronchospasm, d) Eufillin IV, e) Inhaled glucocorticoids.
Answer: d,d

86. was admitted for heartburn, pain in the epigastric region on an empty stomach, relieved by sodium bicarbonate. FEGDS revealed an ulcer (0.5 cm in diameter) in a 12 p.
zuyuschaya function of medium intensity with low alkaline reserves, cholinergic type of reception. Diagnosis: peptic ulcer 12 pc in the acute stage. Choose the most effective and safe drug and determine its dosing regimen:
Answer: Pirenzepine before meals 0.05 g 3 times a day for 2 days, then 0.05 g 2 times a day

87. revealed dyskinesia of the gallbladder hypertonic type. Choose the best treatment option.
Answer: No-shpa 1-2 tablets 3 times a day, immortelle decoction 1/2 cup 30 minutes before meals

88. suffers from chronic cholecystopancreatitis for 5 years. Over the last week after a diet violation, he notes an increase in pain in the right side of the rib cage, nausea, bitterness in the mouth. Choose the most effective choleretic agents that simultaneously have antimicrobial activity:
a) Allochol, b) Cholenzim, c) Nicodin, d) Decoction of tansy, e) Xylitol
Answer: a, c

89. with suicidal intent, she drank 20 tablets of phenazepam. 2 hours after taking the drug, she was taken to the hospital. B-I is conscious, but sharply inhibited. Gastric lavage was performed. Choose the most optimal laxatives: a) Glauber's salt, b) Magnesium sulfate, c) Buckthorn bark extract, d) Bisacodyl,
e) Castor oil, f) Seaweed, g) Vaseline oil
Answer: a,b,d

90. A 46-year-old man was admitted to the cardio intensive care unit with acute transmural myocardial infarction that occurred about 5 hours ago. Appointments: anaprilin 20 mg 4 times a day orally, heparin intravenously at 10,000 units every 4 hours. At the same time, it was possible to achieve an increase in blood clotting time up to 18-23 minutes. On the 4th day, the patient had microhematuria (22 erythrocytes in the field of view). What is your tactic?
Answer: Reduce heparin dose until clotting time is at least 10-12 minutes

91. was admitted with an acute myocardial infarction that occurred 5 hours ago. Appointments: anaprilin 20 mg 4 times a day orally, heparin intravenously at 10,000 units every 4 hours. At the same time, it was possible to achieve an increase in blood clotting time up to 18-23 minutes. The next day, the patient was diagnosed with right-sided lower lobe pneumonia. Benzylpenicillin sodium salt (1,000,000 IU every 4 hours) was prescribed intravenously. After 4 hours, the blood clotting time was 8 minutes. What is your tactic?
Answer: Change the route of administration of penicillin

92. undergone radical surgery for stomach cancer. On the 4th day after the operation, the study of the coagulogram revealed hypercoagulation and a decrease in the fibrinolytic activity of the blood. Is it advisable to prescribe anticoagulants?
Answer: Anticoagulants are indicated, but careful monitoring is necessary to prevent hemorrhagic syndrome.

93. entered the hospital with a complaint of severe weakness, shortness of breath when walking. During examination, anemia (hemoglobin-56 g/l) was revealed in the blood test, the color index was 1.2, when examining the tongue - glossitis. The punctate of the bone marrow revealed a megaloblastic type of hematopoiesis. The concentration of iron in the blood serum was within the normal range. Diagnosis: B 12 -deficiency anamia. Choose the most optimal treatment option.
Answer: Vit.B12 at a dose of 500 mcg / day every other day, folic acid at a dose of 1.5 mg / day, ferrous sulfate (80 mg Fe2 +) 1 time per day

94. after hypothermia, chills occurred, an increase in body temperature to 38.6 ° C, cough with mucopurulent sputum, pain in the right half of the chest. Clinically and radiologically, a diagnosis of right-sided lower lobe pneumonia was established. Treatment was prescribed: cefazolin, 0.5 g 2 times a day IM, hemodez 400 ml IV drip, expectorant mixture of 1 tbsp 6 times a day. Choose an antioxidant drug, the most
effectively influencing the processes of free radical oxidation in the lungs, which should be added to ongoing therapy
Answer: Vit.C

95. due to acute cerebrovascular accident of ischemic type 12 hours ago receives reopoliglyukin 400 ml intravenously drip
1 time per day. Choose the most effective drug in this situation with antioxidant properties.
Answer: Cerebrolysin

96. for 5 years suffers from deforming osteoarthritis of the lower extremities with severe synovitis. Has a history of drug use
allergies (to butadion, heparin, metindol, penicillin, theophylline). In the hospital, reopyrin was prescribed 5 ml intramuscularly 1 time per day, hydrocortisone hemisuccinate 100 mg into the cavity of the knee joints, tavegil 0.001 g 2 times a day. per day. After 3 days, b-noi developed itchy erythematous rashes on the skin of the trunk. What is the most likely cause of the deterioration?
Answer: drug allergic reaction

97. the diagnosis of rheumatoid arthritis was confirmed. What basic drugs for the treatment of rheumatoid arthritis can you prescribe: a) 4,7-chloroquinolone drugs (delagil), b) Cytostatics (azathioprine, cyclophosphamide, etc.), c) Glucocorticoids (prednisolone), d) NSAIDs ,
e) Gold preparations (krizanol), f) Salazopyridazine,
g) Antibiotics (tetracyclines), h) D-penicillamine,
i) Immunomodulators (levamisole)
Answer: a,b,e,f,h,i

98. A patient with rheumatoid arthritis was prescribed methotrexate. How soon will the effect of methotrexate show up?
Answer: After a few months

99. with rheumatoid arthritis was prescribed methotrexate. What measures will you take to monitor the safety of pharmacotherapy with methotrexate in this patient: a) Weekly complete blood count
(preferably 2 times a week), b) Carrying out a blood test to determine the number of platelets every 3-4 weeks,
c) Carrying out a general analysis of urine, d) Determination of the content of uric acid, e) Carrying out a test for occult blood in the feces, f) Determination of the content of transaminases, total bilirubin every 6-8 weeks
Answer: a, b, c, e, f

100. for rheumatism, she receives delagil for a long time. What measures will you take to control the safety of delagil therapy with its long-term use: a) X-ray examination of the chest, g) Determination of the number of platelets, h) Examination of the cornea
Answer: a, b, c, d, e, g, h

101. 39 years old, rheumatoid arthritis, predominantly articular form, 2 degrees of activity. What combination therapy options would be appropriate for this patient?
Answer: Delagil 0.25 g 3 times a day, prednisolone 15 mg / day, krizanol intramuscularly 1 ml of a 5% solution 1 time per week

102. 63 years old suffers from diabetes mellitus, takes glibenclamide. She was admitted to the department with a picture of acute right-sided lower lobe pneumonia, confirmed by X-ray. Chloramphenicol was prescribed, to which the patient had an allergic reaction. -noy revealed a low level of creatinine clearance (24 ml / min), as a result of which ceftriaxone was canceled. Which drug should continue treatment?
Answer: Ciprofloxacin

103. was admitted to the department with a woman for pain in the right mammary gland, an increase in T. to 39.5 C. She fell ill 3 days ago, on the 10th day after birth. Upon admission to the department in the upper outer quadrant of the right mammary
skin hyperemia, a massive infiltrate with a fluctuation in the center. Diagnosis: acute right-sided mastitis. B-naya was operated on. Wound culture taken. Determine antibiotic of first choice.
Answer: Cefazolin

104. entered the department with a picture of acute right-sided mastitis. She fell ill 3 days ago, on the 10th day after childbirth. B-naya operated.
Cefazolin was prescribed. After the 2nd injection of the drug, after 20 minutes, a decrease in blood pressure, dizziness, nausea, vomiting, involuntary urination, and convulsive syndrome appeared. What complication developed in the patient?
Answer: Anaphylactic reaction

105. 21 year old was admitted to the department with a picture of acute right-sided mastitis. She fell ill 3 days ago, on the 10th day after delivery. discharge wounds highlighted
penicillinase-forming staphylococcus aureus and Haemophilus influenzae. Choose an antibacterial drug taking into account the bacterial microflora and pharmacokinetic features
Answer: Levomycetin

106. suffers from chronic tonsillitis and chronic cholecystitis. The examination revealed Staphylococcus aureus, which forms penicillinase, in the culture of the discharged pharynx and in the culture of bile. In the anamnesis, an allergy to oxacillin was noted. Do I need to adjust the dosing regimen of the drug? If yes, how?
Answer: Reduce the frequency of administration and reduce the dose

107. 50 years old complained of general weakness, thirst, frequent urination, itching of the skin and external genitalia. Examination revealed obesity (body weight 96 kg with height 168 cm). Blood glucose 9.9 mmol/l, urine 1 %, the reaction to acetone is negative. What hypoglycemic drugs are optimal in this case?
Answer: biguanides

108. 48 years old, was admitted with complaints of pressing pains that appear during physical activity, which are relieved by nitroglycerin. 3 years ago suffered a myocardial infarction. Vesicular breathing in the lungs. Heart sounds are muffled, systolic murmur at the apex, frequent extrasystoles. Heart rate - 92 per minute. BP - 100/60 mm Hg. Art. The liver is not enlarged, there are no edema. ECG - sinus tachycardia, cicatricial changes in the myocardium, frequent ventricular extrasystoles. Obzidan 160 mg/day, Sustak-Forte 19.2 mg/day, Panangin, Riboxin were prescribed.
What side effect is likely in a patient with this combination of drugs?
Answer: Hypotension, dizziness.

109. Patient M., aged 52, was admitted with complaints of shortness of breath, palpitations, pain in the right hypochondrium, swelling in the legs. For 18 years, he has been in the dispensary with a diagnosis of rheumatism. The skin is pale, acrocyanosis, blush of the cheeks. In the basal parts of the lungs, there are inaudible fine bubbling rales. The boundaries of the relative dullness of the middle are expanded upwards and to the right. Heart sounds are muffled, arrhythmic, systolic murmur at the top, accent?? tone on the pulmonary artery. Pulse-96 per min. Heart rate-140 per min. BP - 130/85 mm Hg. Art. The abdomen is soft, the liver protrudes 3-4 cm from under the edge of the costal arch. Edema on the legs. Daily diuresis -650 ml. ECG: there is no P wave, there are F-F waves, the rhythm is wrong. After intravenous administration of 10 ml of 10% solution of novocainamide: sinus rhythm was restored with a heart rate of 72 per minute, the patient was prescribed novocainamide orally at 0.5 g 4 times a day, digoxin 0.25 mg 1 table.
3 times a day, furosemide 40 mg orally for 3 days. After 5 days, the patient developed nausea, vomiting, diarrhea, dizziness. ECG: sinus rhythm, HR-76 per minute, PQ -0.20 s, QRS-0.1 s. The attending physician canceled digoxin and furosemide and prescribed unithiol and potassium preparations.
Assess the doctor's actions.
Answer: The doctor's actions are correct, since not only is the average daily dose of digoxin exceeded, but there is also an interaction with procainamide for protein binding.

110. Patient D., aged 53, diagnosed with coronary artery disease, stable angina III FC, postinfarction cardiosclerosis, atrial fibrillation, HNK??B Art. He took strophanthin, digoxin, furosemide, panangin in average therapeutic doses. Unexpectedly, the patient's temperature rose to 38.4°C, cough, shortness of breath, crepitus in the lungs on the right appeared. On the radiograph of the lungs on the right in the lower lobe, an area of ​​infiltration is determined. Gentamicin, sulfocamphocaine, suprastin were added to the treatment.
What side effects of treatment are most likely to occur in a patient with such complex therapy?
Answer: When combined with furosemide, the nephrotoxic effect of gentamicin is most likely.

111. A 28-year-old patient was admitted with complaints of palpitations, headache, chills. During the crisis, which develops 2-4 times a year, blood pressure rises to 260/110 mm Hg. Art., heart rate-140 per minute, pale skin, burning pains in the region of the heart, pulsation in the head, sometimes an increase in body temperature up to 38. After attacks of polyuria. In the interictal period, blood pressure is 120/80 mm Hg. Art. An objective study of organic pathology from the internal organs was not detected. In blood and urine tests without pathology.
Specify the most effective drug (first row) for relief of a crisis in a patient:
Answer: Phentolamine.

112. rheumatoid arthritis during the course of treatment with methotrexate, a pronounced nosebleed occurred. What can be caused by: a) Damage to the nasal vessels due to the underlying pathological process, b) Increased PI due to toxic hepatitis caused by methotrexate, c) Increased platelet aggregation under the influence of methotrexate,
d) Drug-induced decrease in the number of platelets, e) Toxic effect of methotrexate on the nasal vessels
Answer: d,d

113. Patient K., 62 years old, 1st degree arterial hypertension. The last deterioration of the condition is due to psycho-emotional stress. On examination: the condition is relatively satisfactory, slight headache. AD-170/100 mm Hg (“working” BP-120/70 mm Hg), heart rate-90 per min. The attending physician prescribed anaprilin 60 mg/day, verapamil 160 mg/day.
What changes can you expect when prescribing verapamil in addition to anaprilin?
Answer: Strengthening the negative dromotropic effect.

114. Patient S., 56 years old, takes nitrosorbide (10 mg) 1m x 4r a day for angina pectoris. How will the tactics of antianginal therapy change if a patient develops a cerebral stroke during treatment with nitrates?
Answer: Cancel nitrates and prescribe an antianginal drug from another group.

115. A 42-year-old patient with chronic glomerulonephritis and arterial hypertension. On admission: BP 200/120 mm Hg, pulse 75-80 beats per minute, swelling on the face, lower back, legs. Total protein in blood serum 3.8 g%, protein in urine 16 g/l. Specify the most effective drugs for antihypertensive therapy in this patient:
Answer: Enalapril.

116. suffering from persistently recurrent bronchial obstruction syndrome, the doctor injected s / c 1 ml of adrenaline. What are the manifestations of toxic
The actions of adrenaline are possible in this situation: a) CNS excitation, b) Extrasystole, c) Toxic damage to the liver, d) Tachycardia, e) Blockade of impulse conduction along the conduction system of the heart.
Answer: a,b,d

117. 57 years old for post-infarction artiosclerosis, congestive heart failure of 2B degree receives 40 mg of furosemide IV and 300 mg
veroshpiron inside. What diuretic therapy will you prescribe to b-nom in case of refractoriness?
Answer: Furosemide 80 mg IV and spironolactone 300 mg orally

118. suffers from non-atopic bronchial asthma, accompanied by profuse bronchorrhea. Pulse 62 min. BP 140/80 mm Hg. After the appointment of atropine sulfate, the b-noi first noted an improvement in the condition - bronchorrhea sharply decreased, but 10 days after the start of treatment, the condition worsened again: fever (37.8 C), shortness of breath, cough with sputum difficult to separate, heart rate 90 per minute appeared. What are the reasons for such changes in the patient's condition?
Answer: Violation of sputum discharge with its subsequent infection

119. A 52-year-old woman suffers from hypertension?? Art. Takes reserpine 1 tab. (0.0001) 3 times a day. BP returned to normal after 1 week. After 4 weeks of regular intake, "hungry" pains appeared in the epigastric region, during gastroscopy, erosive duodenitis was diagnosed. How do you explain its occurrence?
Answer: An increase in the tone of n vagus against the background of reserpine and an increase in gastric secretion.

120. A 60-year-old patient with coronary artery disease, stable angina pectoris IV f.c. Cordarone 600 mg/day was prescribed (as an antianginal drug).
What side effects can be observed in a patient with long-term use of cordarone?
Answer: all of the above

121. When choosing a drug dosing regimen based on T?
determine:
Answer: frequency of reception

122. More accurately characterizes the rate of excretion of drugs from the body:
Answer: total ground clearance

123. The relationship of drugs with plasma proteins:
Answer: determines the possibility of developing side effects when combining drugs

124. The bioavailability value is important to determine:
Answer: routes of drug administration*

125. With prolonged use of strong diuretics, you may experience:
Answer: impaired glucose tolerance

126. Dizziness, lack of sensation in the limbs, difficulty getting up and down without visual control, and other symptoms of toxic effects occur in 75% of patients who:
Answer: receive streptomycin

127. An overdose of sympathomimetics causes:
Answer: rhythm disturbances

128. Adverse reactions associated with the antibiotic moxalactam include the following:
Answer: thrombocytopenia

129. The combined use of indomethacin and gentamicin most often causes:
Answer: kidney dysfunction

130. The use of clavulanic acid in combination with amoxicillin allows:
Answer: expand the spectrum of action of amoxicillin on strains of bacteria that produce beta-lactamase

131. It is planned to additionally prescribe another class 1 antirhythmic drug to a patient receiving diphenin for a long time. Which antiarrhythmic drug will require an increase in dosage by 20-30% from the standard one?
Answer: all drugs

132. Simultaneous ingestion of tetracycline and Ca2 + preparations will contribute to:
Answer: decreased absorption of tetracycline

133. At the same time, the appointment of chloramphenicol and acenocoumarol can lead to:
Answer: to reduce the antibacterial activity of chloramphenicol

134. For heart failure:
Answer: dopamine causes vasoconstriction of the renal cortex in high doses (more than 10 mcg / kg / min)

135. Arterial hypertension is characterized by:
Answer: an increase in the concentration of sodium in the vascular wall

136. Apressin (hydralazine):
Answer: causes tachycardia

137. Beta blockers cause:
Answer: decrease in heart rate

138. The following statements about alpha-blockers are true:
Answer: everything is right

139. Beta-1 - blockers:
Answer: selectively act on beta1-adrenergic receptors, drugs are safe for bronchial asthma

140. Indications for the use of beta-blockers are:
Answer: cardiac arrhythmias

141. Specify the correct statements:
Answer: strophanthin is largely destroyed in the gastrointestinal tract, and therefore its ingestion is irrational

142. Indications for the appointment of SG:
Answer: CNC in patients with ischemic heart disease, postinfarction cardiosclerosis and permanent form of atrial tachyarrhythmia

143. Factor that increases the risk of developing SG intoxication:
Answer: hypokalemia

144. A condition that increases the risk of developing FH intoxication:
Answer: hypothyroidism

145. To reduce the risk of developing nitrate tolerance, you should:
Answer: take breaks between medications

146. In case of development of tolerance to sustac, it can be replaced by:
Answer: corvaton

147. Headache can be caused by:
Answer: Answers A, B, C are correct

148. Similar in mechanism of action to nitroglycerin is:
Answer: molsidomine

149. Overdose of which drugs can cause orthostatic hypotension?
Answer: nitrates

150. Name a group of antiarrhythmics that increase the duration of the action potential:
Answer: potassium channel blockers

151. Which of the following drugs has the most pronounced negative inotropic effect?
Answer: disopyramide

152. Which of the non-cardiac side effects are typical for most class 1C drugs?
Answer: visual impairment

153. During therapy with disopyramide, the following disease may worsen:
Answer: benign prostatic hyperplasia with urinary incontinence

154. When should the dosage of lidocaine be changed from the standard dosage?
Answer: in patients with liver failure

155. Specify the methods of monitoring the effectiveness of diuretics in edematous syndrome:
Answer: everything is right

156. Specify the methods of control over the safety of diuretics in edematous syndrome:
Answer: everything is right

157. Specify an effective and safe way to replenish potassium reserves in the body:
Answer: the appointment of panangin inside 2 tablets 3 times a day

158. Specify the risk factors for side effects of "loop" diuretics:
Answer: daily diuresis more than 3 liters after the introduction of a diuretic

159. Specify the onset of action of spironolactone:
Answer: 4-5 days

160. The following methods are used to alkalinize urine, except:
Answer: potassium citrate 3 mg every 6 hours

161. Urine may be acidic with any of the following drugs except:
Answer: methionine

162. Point out the erroneous positions:
Answer: no erroneous positions

163. Distribute these drugs according to the degree of cumulation:
Answer: neodicoumarin

164. Select the statements that are completely correct for the streptokinase preparation:
Answer: everything is right

165. Select a factor that causes thrombosis or contributes to thrombus formation:
Answer: everything is right

166. Which of the following drugs can reduce the effect of indirect anticoagulants?
Answer: rifampicin

167. What side effects can occur with the use of heparin?
Answer: all of the above

168. A patient with bronchial asthma who has been taking long-acting theophyllines for a long time is prescribed ciprofloxation due to the development of a urinary tract infection. In this case it is necessary:
Answer: reduce theophylline dose by 30%

169. A child receiving carbamazepine for a long time due to the presence of epilepsy develops broncho-obstructive syndrome with respiratory failure of the 2nd tbsp. When prescribing aminophylline to such a patient:
Answer: the dose of aminophylline should be increased by 1.5 times

170. When prescribing theophylline to a smoker:
Answer: dose should be increased

171. Specify the drug that reduces theophylline elimination when administered simultaneously:
Answer: cimetidine

172. A patient with bronchial asthma, who received teotard for a long time, developed nausea, vomiting, headaches, and insomnia against the background of influenza infection and fever. Meningeal symptoms are negative. Therapeutic tactics in this case:
Answer: stop teotard or reduce its dose by 50%

173. Side effects of theophylline may include the following except:
Answer: development of edematous syndrome

174. A specific side effect that occurs when using theophylline in children of the 1st year of life is:
Answer: melena

175. Specify the inhaled glucocorticosteroid drug with the lowest bioavailability:
Answer: fluticasone propionate

176. Designate an inhaled glucocorticosteroid drug that has the lowest affinity for glucocorticosteroid receptors in the human lungs:
Answer: fluticasone propionate

177. Specify the drug with the highest degree of safety (according to the safety index:)
Answer: prednisolone

178. Which of the glucocorticosteroid drugs most contributes to the development of myopathy?
Answer: triamcinolone

179. Slowing down the excretion of sodium and water from the body, increased excretion of potassium (mineralocorticoid effect) is more characteristic of:
Answer: hydrocortisone

180. Mineralocorticoid activity is absent in:
Answer: dexamethasone

181. Choose the correct answer. Glucocorticoids:
Answer: are contraceptive hormones

182. When conducting pulse therapy, it is more preferable:
Answer: methylprednisolone

183. When prescribing for a long time, it is preferable to use:
Answer: prednisolone

184. What H1-histamine receptor blocker is contraindicated in anaphylactic shock?
Answer: diphenhydramine (diphenhydramine)

185. Choose the optimal H1-histamine receptor blocker for the treatment of allergic rhinitis:
Answer: azelastine (allergodil)

186. Designate a drug from the group of mast cell membrane stabilizers in the dosage form in the form of a powder for inhalation:
Answer: cromoglycic acid (bicromate)

187. An immunostimulant of microbial origin includes:
Answer: ribomunil

188. The main indication for the appointment of ribomunil is:
Answer: prevention of recurrent infections of the upper respiratory tract

189. The following antibacterial drugs penetrate the blood-brain barrier well:
Answer: 3rd generation cephalosporins

190. The new generation of macrolide antibiotics has the following advantages except:
Answer: renal route of excretion

191. Fluoroquinolones differ from quinolones in the following ways except:
Answer: bacteriostatic action

192. Which statements about cephalosporins are correct?
Answer: everything is right

193. The consequences of taking antibiotics include:
Answer: everything is right

194. Specify the drug of choice for urinary tract infection caused by Pseudomonas aeruginosa:
Answer: ceftazidime

195. What drugs are indicated for the treatment of chlamydial infection of the genitourinary tract:
Answer: rovamycin

196. Specify the drug with the least favorable pharmacokinetic characteristics:
Answer: ketoconazole

197. Specify an antimycotic drug that is not metabolized in the liver:
Answer: fluconazole

198. Specify an antimycotic drug (from the group of allylamines) used primarily for the treatment of dermatomycosis:
Answer: terbinafine

199. Specify the clinical condition that is an indication for NSAID monotherapy:
Answer: extra-articular rheumatic diseases (myositis, tendovaginitis, synovitis)

200. Acetylsalicylic acid is characterized by:
Answer: when taken orally, it is absorbed mainly from the upper small intestine

201. In comparison with indomethacin, acetylsalicylic acid has more pronounced:
Answer: antiplatelet effect on platelets

202. The rate of excretion of acetylsalicylic acid and its metabolites is affected by:
Answer: urine pH level

203. Gastrointestinal complications when using acetylsalicylic acid are associated with:
Answer: all of the above

204. Phenylbutazone is characterized by:
Answer: everything is right

205. When indomethacin interacts with other drugs:
Answer: diuretic activity of furosemide decreases

206. What adverse reactions of NSAIDs are corrected by the complex preparation Arthrotec (diclofenac sodium + misoprostol)
Answer: NSAID gastropathy

207. What characteristics of paracetamol put this drug in first place among analgesics-antipyretics?
Answer: earlier onset of analgesic and antipyretic effect

208. Choose a drug that selectively inhibits cyclooxygenase2:
Answer: meloxicam

209. The best analgesic effect of fentanyl is observed in combination with:
Answer: droperidol

210. Name an anti-inflammatory agent of prolonged action:
Answer: piroxicam

211. A type 2 hypertensive crisis with a heart rate of 62 in min. BP 200/140 mmHg developed in B-oh 52l.
Answer: Furosemide

212. has been suffering from type 1 diabetes mellitus for 6 years, receives insulin at 54 U / day, which maintains the level of glycemia within 7.0 mmol / l. Recently, due to an increase in blood pressure to 16090 mmHg. the attending physician prescribed hypothiazide at a daily dose of 75 mg in combination with enalapril at a dose of 5 mg. After 10 days, the patient's blood sugar level was 10.5 mmol, and he felt worse. What is the leading cause of changes in blood sugar levels?
Answer: Combination of enalapril with hypothiazide

213. a convulsive form of a hypertensive crisis developed, the condition is severe, the numbers of blood pressure are 200120 mm Hg, heart rate is 120 per minute. Which drug should be used to start therapy?
Answer: diazepam

214. against the background of the use of the antibiotic ceftriaxone for 10 days, a picture of pseudomembranous colitis developed. What is the first step of the medical care algorithm?
Answer: withdrawal of ceftriaxone, administration of vancomycin or metronidazole

215. due to exacerbation of gastric ulcer, clarithromycin was prescribed in the complex of therapy. Name the main distinguishing features of the drug from erythromycin.
Answer: everything is right

216. after surgery on the abdominal cavity on the 4th day left-sided lower lobe pneumonia developed. The results of express analysis showed the presence of MRSA, penicillin- and aminoglycoside-resistant strains of enterococci. Drugs of choice:
Answer: Vancomycin

217. is in the intensive care unit for Pseudomonas aeruginosa. Choose 1st line drugs for treatment?
Answer: Ceftazidime + aminoglycosides

218. 40 years without concomitant diseases due to community-acquired pneumonia on an outpatient basis, spiramycin was prescribed orally at 3 million IU 2 pc, on the 2nd day of treatment, intense gastralgia, nausea, and single vomiting were noted. Choose an alternative drug.
Answer: doxycycline

219. with chronic obstructive bronchitis, pneumonia of moderate severity was detected, amoxiclav was prescribed orally at 625 mg 3 rs on an outpatient basis. On the 2nd day, the patient developed urticaria, bronchospasm. Name an alternative drug for the treatment of pneumonia.
Answer: moxifloxacin by mouth

220. A 44-year-old HIV-infected patient was diagnosed with pneumocystis pneumonia. Name the drug for treatment?
Answer: co-trimoxazole IV 20 mgkgs 4 rs for 21 days

221. Have b-Noah 28l. daily symptoms of bronchial asthma, frequent exacerbations, frequent nocturnal symptoms are noted, severe persistent bronchial asthma is diagnosed. Name the drugs of basic therapy.
Answer: inhaled glucocorticoids (more than 1000 micrograms of beclomethasone dipropionate) + long-acting inhaled beta-2-agonists

222. A pregnant woman (6-7 weeks gestation) came to the doctor with clinical signs of acute pneumonia. What groups of antibacterial drugs are allowed for use in pregnant women?
Answer: cephalosporins

223. A 57-year-old patient is treated with an ACE inhibitor, enalapril, for moderate arterial hypertension. In a patient after 2 years of taking the drug, there is a lack of effect. What is the most appropriate option for optimizing therapy?
Answer: adding a diuretic (hypothiazide or indapamide) to the drug

224. receives an antibacterial drug for an infectious process. With intravenous infusion of the drug, a reaction is noted in the form of a pronounced reddening of the skin of the upper half of the trunk, face, neck, the symptoms decrease significantly with a decrease in the infusion rate. What drug has such a reaction?
Answer: Vancomycin

225. A pregnant woman has an activation of the rheumatic process. Which drug from the group of anticoagulants can be prescribed to a pregnant woman?
Answer: Heparin

226. was admitted with f-mi to increase blood pressure to the numbers 15090 mm Hg. against the background of psycho-emotional overstrain, palpitations, anxiety, sleep disturbance. A year ago, she was diagnosed with type 2 diabetes mellitus and receives maninil. Name the drug of choice for the treatment of hypertension.
Answer: Atenolol

227. in connection with the symptoms of angina pectoris and rhythm disturbances, drugs were prescribed: anaprilin 200 mgs and verapamil 240 mgs for a long time. What are the possible side effects?
Answer: Development of a-b blocks, bradycardia

228. A 34-year-old woman is taking estrogen-containing contraceptives. The attending physician prescribed doxycycline at a dose of 200 mgs for 2 weeks. What is the likely interaction expected?
Answer: Decreased effect of contraception

229. The anesthesiologist prescribed diazepam for the prophylactic purpose of ketamine administration. What condition is being prevented in this way?
Answer: Postanesthesia hallucinations

230. 46 years old was admitted with a picture of acute destructive appendicitis. Drug of choice for antibiotic prophylaxis?
Answer: Cefazolin

231. A patient came to the doctor with complaints of cough, fever up to 39 C, chest pain. Right-sided bronchopneumonia was diagnosed. A drug was prescribed for 3 days, which has a post-antibiotic effect. Name the drug of choice.
Answer: Azithromycin

232. due to acute right-sided pyelonephritis, cefazolin was prescribed at a dose of 2 gs for 10 days. What is the most common mistake when choosing this antibiotic?
Answer: Insufficiently high activity against gram-negative flora

233. simultaneously receives a fluoroquinolone antibiotic - ofloxacin for urinary tract infection and diclofenac sodium for articular syndrome for 14 days. What is the likely interaction expected?
Answer: increased risk of CNS excitation and seizures

234. after hypothermia, chills occurred, an increase in body temperature to 38.6 ° C, a cough with mucopurulent sputum, pain in the left half of the chest. Clinically and radiologically, a diagnosis of left-sided lower lobe pneumonia was established. per day IM, hemodez 400 ml IV drip, expectorant mixture 1st.l.6r. per day. On the 3rd day there was a reaction in the form of urticaria, skin itching. Choose an antibacterial drug to replace?
Answer: Spiramycin

235. During anesthesia, the anesthesiologist prescribed an antibacterial drug for the purpose of antibiotic prophylaxis. The patient developed respiratory arrest. What drug was used?
Answer: Gentamicin

236. for 10 years suffers from deforming osteoarthritis of the lower extremities with severe synovitis. She has a history of drug allergy to butadione. Reopirin 5 ml intramuscularly 1 time per day was prescribed in the b-noy department. A day later, b-noy developed itchy erythematous rashes on the skin of the trunk. What is the most likely cause of the deterioration?
Answer: drug allergic reaction

237. Novocainamide was prescribed for cardiac arrhythmias, while cetirizine was prescribed for seasonal allergic rhinitis. What is the likely interaction expected?
Answer: severe forms of arrhythmia (pirouette type)

238. A patient with SLE was prescribed methotrexate. How long does it take to see a stable therapeutic effect?
Answer: birneshe aydan kayin

239. with a hypertensive crisis, fosinopril was prescribed. Despite taking an adequate dose of the drug, there was no decrease in blood pressure in the next minutes and hours. Name the reason.
Answer: Depot drugs are not used in crises

240. for the purpose of planned therapy of hypertension, apressin was prescribed for a long time. After a month of using the drug, the patient began to experience palpitations, angina pectoris, and a decrease in the effect of treatment. Name the main cause of the developed phenomena
Answer: Apressin is not prescribed for planned therapy of hypertension

241. At the age of 42, he was admitted to the intensive care unit for a severe hypertensive crisis. Sodium nitroprusside was administered intravenously for 5 days. On the 6th day, the patient developed a picture of intoxication in the form of indomitable vomiting, a decrease in the function of the cardiovascular, respiratory and excretory systems. Name the main cause of the developed condition.
Answer: Drug overdose (accumulation of thiocyanates in the blood)

242. 54 years old suffers from type 1 diabetes mellitus, takes prolonged insulin. She was admitted to the department with a picture of acute right-sided lower lobe pneumonia, confirmed by X-ray. Ceftriaxone was prescribed, to which the patient had an allergic reaction. However, the examination revealed a low level of creatinine clearance (30 ml/min) in B-noi, as a result of which amikacin was canceled. Which drug should continue treatment?
Answer: Spiramycin

243. was admitted to the department with a woman for pain in the lower abdomen, an increase in T. to 39.5 C. She fell ill 2 days ago, on the 6th day after birth. Gynecological examination showed a picture of acute postpartum endometritis. The results of bakposev: Staphylococcus aureus, forming penicillinase, Proteus. Determine antibiotic of first choice
Answer: cefepime

244. 25 liters was admitted to the department with a picture of acute right-sided pyelonephritis. She fell ill 3 days ago after hypothermia. Cefazolin was prescribed. After the 2nd injection of the drug, after 10 minutes, a decrease in blood pressure, dizziness, nausea, vomiting, involuntary urination, and convulsions appeared. What complication developed in the patient?
Answer: Anaphylactic reaction

245. A 28-year-old patient was admitted to the department with a picture of acute right-sided mastitis. She fell ill on the 12th day after delivery. , which forms penicillinase, and candida. Choose an antibacterial drug taking into account the bacterial microflora and pharmacokinetic features
Answer: Oxacillin + fluconazole

246. suffers from chronic cholecystitis. In the bile culture during the examination, Staphylococcus aureus and Escherichia coli were revealed. She has a history of allergy to oxacillin. Drugs of choice.
Answer: Ceftriaxone

247. 58 years old complained of general weakness, thirst, frequent urination, itching of the skin and external genitalia. On examination: body weight 56 ​​kg with height 168 cm). Blood glucose 12.3 mmol/l, urine 1.5 %, the reaction to acetone is negative. What hypoglycemic drugs are optimal in this case?
Answer: Sulfonylureas

248. 53 years old, was admitted with complaints of palpitations, interruptions, and sometimes shortness of breath. These phenomena began to disturb after myocardial infarction 2 years ago. Taking novocainamide for 3 months brought significant relief. However, in recent years, the state of health has worsened. Further management of the patient.
Answer: complete examination and selection of the drug

249. A 33-year-old patient is receiving a combination of drugs for postoperative purulent peritonitis: ceftriaxone + amikacin + metronidazole. She has a history of gallstone disease. Name the drug that should not be prescribed to the patient.
Answer: Ceftriaxone

250. A 45-year-old woman with candidal pneumonia was prescribed fluconazole IV for 3 days, then orally. On the 4th day of treatment, a reaction was noted in the form of severe headaches, nausea. It was decided to replace the drug with ketoconazole. Assess the adequacy of tactics.
Answer: Ketoconazole is not an adequate substitute due to unfavorable pharmacokinetic characteristics

251. 42 years old, was admitted with complaints of severe palpitations, an increase in blood pressure to 240140 mm Hg. Examination revealed a significant increase in the level of catecholamines in the blood. Name the drug of choice for stopping the crisis.
Answer: phentolamine

252. 50 years old, was admitted with a picture of acute left-sided pyelonephritis. Cefazolin + gentamicin were prescribed in medium therapeutic doses. During the examination, the patient's creatinine clearance was 50 ml min. What are the likely consequences of therapy?
Answer: Risk of nephrotoxicity

253. 48 years old with paroxysmal supraventricular and ventricular tachycardia was prescribed cordarone IV on the first day, then inside. Examination revealed a dysfunction of the thyroid gland, a blockade of 2-3 degrees. Further tactics.
Answer: Cancellation of the drug, the appointment of novocainamide

254. For 50 years, he has been in the cardio intensive care unit for acute myocardial infarction, receiving complex therapy. What are the main monitoring parameters when prescribing direct anticoagulants?
Answer: APTT, blood clotting time, urine for erythrocytes

255. A patient is in the department after surgery for appendicitis. The attending physician prescribed gentamicin 80 mg 3 times a day. The patient had acute glomerulonephritis 2 years ago, currently creatinine clearance is 50 ml min. What correction is required?
Answer: replacement with a drug that does not have nephrotoxicity

256. 56 years old, receives digoxin at 0.25 gs during the last year. At present, an increase in blood pressure figures to 180110 mm Hg has begun to be noted. The attending physician prescribed lisinopril at a dose of 10 mgs. After 3 months, the patient was diagnosed with digitalis intoxication. Further tactics.
Answer: prescribing another antihypertensive drug

257. A 53-year-old was diagnosed with candidal and aspergillus meningitis. Drugs of choice.
Answer: Amphotericin B

258. A 58-year-old patient receiving glucocorticoids for a long time, a broad-spectrum synthetic antibiotic was included in the therapy complex due to an infection of the reproductive area. On the 14th day of joint use, a severe complication was noted in the form of a rupture of the Achilles tendon. Name the antibiotic that, in combination with corticosteroids, caused this complication.
Answer: Levofloxacin

259. B.43 years old was admitted to the department with a picture of acute chlamydial pneumonia. Name the drugs of choice.
Answer: Rovamycin

260. A 24-year-old patient was diagnosed with an uncomplicated form of gonorrhea. Name the drug of choice.
Answer: Ceftriaxone

261. Choose the correct statement: a) bioavailability is the amount of drugs entering the systemic circulation, expressed as a percentage of the administered dose, b) bioavailability is determined by the amount of drug adsorption in the gastrointestinal tract and expressed
the effect of the first passage through the liver. c) bioavailability is determined by the formula: F \u003d AUC (in / m or orally) / AUC (in / in). d) the bioavailability of drugs when administered intramuscularly is determined by the degree of its absorption and biotransformation in the body.
Answer: a B C

262. suffers from non-atopic bronchial asthma, accompanied by profuse bronchorrhea. Pulse 62 min. BP 140/80 mm Hg. Which drugs are more preferable?
Answer: Atrovent

263. Mechanisms of absorption of drugs in the intestine:
a) passive diffusion, b) filtration, c) active transport, d) facilitated transport, e) pinocytosis:
Answer: a, d

264. was admitted for heartburn, pain in the epigastric region on an empty stomach, relieved by sodium bicarbonate. FEGDS revealed an ulcer (0.5 cm in diameter) in a 12 p.
zuyuschaya function of medium intensity with low alkaline reserves, cholinergic type of reception. Diagnosis: peptic ulcer 12 pc in the acute stage. Choose the most effective and safe drug and determine its dosing regimen:
Answer: Pirenzepine before meals 0.05 g 3 times a day for 2 days, then 0.05 g 2 times a day

265. revealed dyskinesia of the gallbladder hypertonic type. Choose the best treatment option.
Answer: No-shpa 1-2 tablets 3 times a day, immortelle decoction 1/2 cup 30 minutes before meals

266. suffers from chronic cholecystopancreatitis for 5 years. Over the last week after a diet violation, he notes an increase in pain in the right side of the rib cage, nausea, bitterness in the mouth. Choose the most effective choleretic agents that simultaneously have antimicrobial activity:
a) Allohol, b) Cholenzim, c) Nicodin, d) Decoction of tansy,
e) Xylitol
Answer: a, c

267. with suicidal intent, she drank 20 tablets of phenazepam. 2 hours after taking the drug, she was taken to the hospital. B-I is conscious, but sharply inhibited. Gastric lavage was performed. Choose the most optimal laxatives: a) Glauber's salt, b) Magnesium sulfate, c) Buckthorn bark extract, d) Bisacodyl,
e) Castor oil, f) Seaweed, g) Vaseline oil
Answer: a,b,d

268. A 46-year-old man was admitted to the cardio intensive care unit with acute transmural myocardial infarction that occurred about 5 hours ago. Appointments: anaprilin 20 mg 4 times a day orally, heparin intravenously at 10,000 units every 4 hours. At the same time, it was possible to achieve an increase in blood clotting time up to 18-23 minutes. On the 4th day, the patient had microhematuria (22 erythrocytes in the field of view). What is your tactic?
Answer: Reduce heparin dose until clotting time is at least 10-12 minutes

CATEGORIES

POPULAR ARTICLES

2023 "kingad.ru" - ultrasound examination of human organs